Download as docx, pdf, or txt
Download as docx, pdf, or txt
You are on page 1of 70

Torts outline

Overview............................................................................................................................................................................... 3
A. An introduction to Torts:..........................................................................................................................................3
(i) What is a Tort?.........................................................................................................................................................3
(ii) An Example of a Tort Suit......................................................................................................................................3
(iii) Overview...............................................................................................................................................................3
Negligence: Liability for Physical Harms...........................................................................................................................4
A. The Duty Element......................................................................................................................................................4
(i) Negligence: A Brief Overview.................................................................................................................................4
(ii) The Duty Element and the General Duty of Reasonable Care................................................................................4
(iii) Qualified Duties of Care........................................................................................................................................6
(a) Affirmative Duties...........................................................................................................................................6
(b) Premises Liability............................................................................................................................................7
(c) Pure Economic Loss........................................................................................................................................8
B. The Breach Element...................................................................................................................................................9
(i) Duty, Breach, and the Meaning of “Negligence”.....................................................................................................9
(a) Ordinary Care..................................................................................................................................................9
(b) Extraordinary Standard of Care.......................................................................................................................9
(c) Strict Liability................................................................................................................................................10
(d) Negligence.....................................................................................................................................................10
(ii) Defining the Person of Ordinary Prudence...........................................................................................................11
(iii) Industry and Professional Custom.......................................................................................................................12
(a) Custom/ The TJ Hooper Rule........................................................................................................................12
(b) Medical Malpractice......................................................................................................................................12
(c) Informed consent...........................................................................................................................................13
(iv) Reasonableness, Balancing, and Cost-Benefit Analysis.......................................................................................14
(a) The Hand Formula.........................................................................................................................................14
(v) Proving Breach: Res Ipsa Loquitur.......................................................................................................................15
C. The Causation Element............................................................................................................................................16
(i) Key Terms and Concepts.......................................................................................................................................16
(ii) Proving Actual Causation Under the Preponderance Standard.............................................................................16
(iii) Multiple Necessary and Multiple Sufficient Causes............................................................................................17
(a) multiple necessary causes..............................................................................................................................18
(b) multiple sufficient causes...............................................................................................................................18
(c) NESS............................................................................................................................................................. 18
(iv) Causation and Tortfeasor Identification...............................................................................................................19
(a) Alternative causation.....................................................................................................................................19
(b) Market share liability.....................................................................................................................................19
D. Aligning the Elements: Proximate Cause and Palsgraf.........................................................................................20
(i) Proximate Cause....................................................................................................................................................20
(a) Natural and ordinary test (no longer used).....................................................................................................20
(b) Directness Test (no longer used)....................................................................................................................21
(c) Foreseeability Test (Current Test).................................................................................................................21
(d) Scope of Risk Test.........................................................................................................................................21
(ii) The Relational Aspect of Breach of Duty: Palsgraf..............................................................................................22
(iii) Superseding Cause and Affirmative Duties.........................................................................................................22
E. Statutory Supplements.............................................................................................................................................23
(i) Negligence Per Se..................................................................................................................................................23
F. Defenses.....................................................................................................................................................................24
(i) Contributory Negligence and Comparative Responsibility....................................................................................24
(a) Contributory Negligence................................................................................................................................24
(b) Comparative Fault/ Comparative Responsibility...........................................................................................24
(c) Divided Damages...........................................................................................................................................25
(ii) Assumption of Risk..............................................................................................................................................25
(a) Express assumption of risk............................................................................................................................25
(b) Implied assumption of risk.............................................................................................................................26
(iii) Immunities and Exemptions from Liability.........................................................................................................27
(a) Intra-familial and charitable immunities........................................................................................................27
(b) Governmental/ Sovereign immunity..............................................................................................................27
(c) Liability Exemptions: No Duty Rules for Local Government & Private Entities...........................................28
1. Local Government..........................................................................................................................................28
2. Private Entities...............................................................................................................................................28
G. Damages and Apportionment.................................................................................................................................29
(i) Elements and Availability of Damages..................................................................................................................29
(a) Compensatory Damages................................................................................................................................29
(b) Punitive Damages..........................................................................................................................................30
Battery, Assault, and Infliction of Emotional Distress......................................................................................................31
A. Battery and Assault..................................................................................................................................................31
(i) Battery...................................................................................................................................................................31
(a) Battery: Prima Facie Case..............................................................................................................................31
(b) Harmful or offensive touching.......................................................................................................................31
(c) Intent..............................................................................................................................................................32
(ii) Assault.................................................................................................................................................................. 33
(a) Assault: Prima Facie Case.............................................................................................................................33
(b) Reasonable apprehension...............................................................................................................................33
(c) Words Alone do not suffice for a successful Assault Claim..........................................................................33
(d) Threats + Conduct Can be Assault; No Bodily Contact is Necessary............................................................33
(iii) Transferred Intent................................................................................................................................................34
(iv) Standard Defenses to Battery and Assault............................................................................................................34
(a) Consent..........................................................................................................................................................34
(b) Self-Defense and Defense of Others..............................................................................................................35
(c) Defense and Recapture of Property................................................................................................................36
B. Infliction of Emotional Distress...............................................................................................................................36
(i) Intentional Infliction of Emotional Distress (IIED)................................................................................................36
(ii) Negligent Infliction of Emotional Distress............................................................................................................38
(a) Physical impact rule.......................................................................................................................................38
(b) Zone of danger test........................................................................................................................................38
Liability without Fault and Products Liability..................................................................................................................40
A. Property Torts and Ultrahazardous Activities.......................................................................................................40
(i) Property Torts........................................................................................................................................................40
(a) Trespass to land: prima facie case..................................................................................................................40
(b) Trespass and necessity...................................................................................................................................40
(c) Conversion and trespass to chattel (T/C).......................................................................................................41
1. Trespass to chattel (T/C) (NOT ON EXAM).............................................................................................41
2. Conversion................................................................................................................................................41
3. T/C versus Conversion..............................................................................................................................42
(d) Consent (with notes on media trespass and on defenses other than consent).................................................42
(e) Nuisance....................................................................................................................................................43
(ii) Ultrahazardous (Abnormally Dangerous) Activities.............................................................................................44
B. Products Liability.....................................................................................................................................................46
(i) Introduction............................................................................................................................................................46
2
(a) Precursors......................................................................................................................................................46
(ii) Basics of a Products Liability Claim.....................................................................................................................46
(a) What counts as an injury?..............................................................................................................................47
(b) What is a “product”?......................................................................................................................................47
(c) Who or what is a “seller”?.............................................................................................................................47
(d) The key to products liability: defect...............................................................................................................47
1. Manufacturing defect.................................................................................................................................47
2. Design defect.............................................................................................................................................47
3. Failure to warn or instruct..........................................................................................................................48

Overview
A. An introduction to Torts:
(i) What is a Tort?
 What is meant by "tort," "torts,", & "tort law"?
o "to commit a tort is to act in a manner that the law deems wrongful toward and injurious to another, such
that the other gains a right to bring a lawsuit to obtain relief from the wrongdoer (or tortfeasor)"
 "The word 'torts' in turn refers to a collection of named and relatively well-defined legal wrongs
that, when committed, generate a right of action in the victim against the wrongdoer"
 "Tort law consists of the rules and principles that define wrongful conduct, delineate the
circumstances under which a victim can obtain redress, and designate the form that such redress
may take."
 Main topics in the course
o Negligence o Property torts (including trespass and
o Battery nuisance)
o Assault o Ultrahazardous activities
o Infliction of emotional distress o Products liability
 Every state has its own tort law
 Restatement of torts
o In some areas of law, ideas similar enough--> consensus
 Other areas states take dramatically different approaches
o Not binding, don't have the force of law
 The basic concept of negligence as a tort
o A failure to heed a duty of reasonable care that causes an injury to a person to whom that duty is owed
 Causation
o " that there be some reasonable connection between the act or omission of the defendant and the damage
which the plaintiff has suffered"

(ii) An Example of a Tort Suit


 Walter v. Wal-Mart
o Facts: Walter went to Wal-mart for cancer medication. The pharmacist gave her a much stronger drug,
which gave her adverse effects
o Held: Wal-Mart is Liable
o To establish that she was a victim of Wal-Mart's negligence, Walter had to prove:
 (1) Wal-Mart owed her a duty of reasonable care;

3
 (2) It breached that duty;
 (3) the breach of duty caused her to suffer adverse effects; and
 (4) these effects are recognized by the law as an injury

(iii) Overview
 What is a tort?
o Civil wrongs for which victims have causes of action against wrongdoers to recover money judgements
o Without a harm there is no tort, doesn’t matter if there’s a harm if can’t make out a tort
 Goals of the tort system
o Compensation (make injured P whole)
o Corrective justice
o Deterrence
 Objectives of tort law
o Fairness/ corrective justice perspective
 Achieve fairness for its own sake
o Instrumental/safety perspective
 Incentivize actors to behave more carefully in the future, emphasis on deterring future losses
 Theories of liability
o Intentional torts
 P arguing D should be liable because he intentionally caused harm
 Likely to get punitive damages
o Negligence torts
 P arguing D failed to exercise reasonable care
o Strict liability (liability without fault)
 Restricted classes of cases where if D did activity and someone got hurt, even if didn’t do it
negligently or were careful, D is liable
 Types of damages
o Nominal
 Token/ symbolic amount awarded simply to commemorate the P’s vindication in court
o Compensatory
 Compensate P for losses caused by D’s tortious conduct, make whole (same position as were
before) with money
o Punitive damages
o Punish D for particularly outrageous tortious conduct
 Civil lawsuit trial
o P’s prima facie case
 P bears burden of proof
 P must prove elements of c/a
 P must convince the jury that it is more probable than not that his injuries are due to D’s
negligence
o D’s defense
 D can negate one or more elements
 D bears burden of proof for affirmative defenses
o Judge/ jury
 Apply law to facts
4
Negligence: Liability for Physical Harms
A. The Duty Element
(i) Negligence: A Brief Overview
 Negligence: conduct negligently causing damages, dominant tort theory, conduct falls below the standard of what
a reasonable person would do under similar circumstances to protect other against unreasonable risk of harm
o About conduct not INTENT was the D’s conduct unreasonably risky?
 Elements of a prima facie case
o To establish that the P is a victim of D’s negligence, P has to prove:
Element Description
(1) Duty General duty to use reasonable care toward foreseeable others

(2) Breach of D failed to use reasonable care


Duty
(3) Causation  Actual causation  D’s conduct was cause in fact of P’s harm
 Proximate Cause the harm was within the scope of liability

(4) Damages/ P must suffer actual, tangible harm (no nominal damages)
Harm

o The D can negate any one of these elements to win a case


 Some negligence basics
o "negligence in tort law is a failure to heed a duty of care owed to another that causes injury to that other"
o "because of its breadth, negligence is today the tort most commonly relied upon by accident victims to obtain
redress from an alleged wrongdoer"
o "suits for injuries caused by careless driving, incompetently rendered professional services, and unsafe
premises are usually suits for negligence"
 What counts as injury for negligence?
o Physical harm, whether:
 Bodily harms; or
 Damage to property, including land, structures, personal possessions
o Economic loss;
o Emotional distress

(ii) The Duty Element and the General Duty of Reasonable Care
 GENERAL RULE: everyone owes a duty to use reasonable care toward foreseeable others
o Duty is a matter of policy for a judge to decide, use it as a mechanism of limiting liability
o No duty is imposed on a person to take precautions against events that cannot reasonably be foreseen
 No general duty to act: a person generally cannot be liable in tort solely on the grounds that she has failed to act
o Exceptions
 Special relationships
 D involved in injury
 D and V as co-venturers
 Assumption of duty
 Duty to control others
 Old Cases
5
o Winterbottom
 Leading case for arguing that manufacturers aren’t liable to buyers who they don’t directly sell to
 Facts: This was the early English case about the carriages. Wright builds, maintains, and sells
carriages, and sells one to the London Postmaster General. Winterbottom is an employee of the
Postmaster – there was no pre-existing relationship between Winterbottom and Wright. The
carriage crashed, and Winterbottom claimed a lack of soundness of the carriages
 Held: rejected Winterbottom’s argument
 b/c there was no Privity between the parties – only the parties and the Postmaster
o Carriage manufacturer not liable to carriage drivers because there is no privity of
K
o Heaven v pender
 Facts: An owner of a dry dock supplied ropes that supported a stage slung over the side of a ship.
The stage failed because the supplied ropes had been previously burned. The failure of the stage
injured an employee of an independent contractor working in the dry dock. The dry dock owner,
the defendant, had failed in his duty of care to give reasonably careful attention to the condition
of the ropes, prior to employing them to hold up the stage. The defendant was found liable.
 Held: A duty of reasonable care attaches whenever it is reasonably foreseeable that careless
conduct on one’s party may risk physical harm to persons such as the P
 Broader standard than Winterbottom
 But confines this duty to immediate buyer/user
 Court distinguishes Winterbottom – says no liability there because of remoteness of
injury from manufacturer’s careless acts
 Remoteness defined in terms of time and space
o Winchester
 When the nature of the business is selling goods to third parties, manufactures owe a duty to them
when the good can create an imminent danger if improperly manufactured
 Facts: This was the English case about the mislabeled bottle of poison, sold as if it was medicine
– the manufacturer sold it to a pharmacist who sold it to the plaintiff’s husband
 Held: The Court rejected the WINTERBOTTOM argument, drawing a distinction on the grounds
that creating a poison carries great risk of death or bodily harm, which is the inevitable injury
from the ingestion of poison
 It is not a rejection of WINTERBOTTOM, but creates an exception
 General Duty of Reasonable Care
o Macpherson
 There is a duty rooted in the law for auto manufacturers
 Manufacturers invite people to their products, and the law to take into account the costs
they impose on others
 Facts: E is a manufacturer of automobiles. It sold an automobile to a retail dealer. The retail
dealer resold to the plaintiff. While the P was in the car, it suddenly collapsed. He was thrown out
and injured. One of the wheels was made of defective wood, and its spokes crumbled into
fragments. The wheel was not made by the defendant; it was bought from another manufacturer.
There is evidence, however, that its defects could have been discovered by reasonable inspection,
and that inspection was omitted.
 Held: here is a duty and it is rooted in the law (rather than a contract)
 Cardozo’s options:

6
o (1) Could say there is no duty because no privity (Winterbottom)
o (2) Could say no privity, but find duty by extending Thomas v. Winchester
exception
o (3) Could say forget the privity requirement, and follow Brett’s Heaven
formulation
 Cardozo takes 2 and 3 – unclear if extending Winchester to anything that
is reasonably certain to be dangerous if improperly made (regardless of
contract), or junking the Winchester principle entirely
 See emergence of the tendency to try to think conceptually about duty
o Buick cites to Winterbottom – says not liable
o Cardozo instead relies on Heaven and Thomas v. Winchester
 Buick could argue that car isn’t inherently dangerous/doesn’t represent
an imminent danger – would be up to jury
 Typical policy question underlying tort suits  is allowing tort suits the best way to
regulate automotive manufacturing safety?
o Cardozo says manufacturers invite people to use their products, and the law has
to take into account the costs they impose on others
 Also the notion of imminence has changed along with changes in
commerce
o Mussivand v David
 Facts: Mussivand sued David because he got an STD from sleeping with David’s wife, West
 Held: A person who knows, or should know, that he or she is infected with a venereal disease has
the duty to abstain from sexual conduct or, at a minimum, to warn those persons with whom he or
she expects to have sexual relations of his or her condition.
 A spouse is a foreseeable sexual partner and a person who has a venereal disease who
fails to inform a married person with whom he or she is engaging in sexual contact of his
or her condition is liable to the third-party spouse until the initially infected spouse
knows or should have known he or she is infected with a venereal disease.

(iii) Qualified Duties of Care


 situations in which courts hold that D owes P less than this regular duty
 Affirmative Duties to Rescue and Protect
o Areas where we find qualified duties of care
 Cases where plaintiff is alleged to have:
1. Unreasonably failed to provide assistance or protection to the plaintiff (affirmative duty or duty-
to-rescue or nonfeasance cases);
2. Permitted or maintained unreasonably dangerous conditions on property in his or her possession
(premises liability cases);
3. Acted without reasonable care for the plaintiff’s economic prospects (pure economic loss cases);
4. Negligently inflicted emotional distress.

(a) Affirmative Duties


o There is no general duty to protect or rescue (i.e. to give aid/assistance to another who faces the prospect
of harm from a source other than oneself)
 Osterlind v Hill

7
 Facts: This was the case of the man who operated the canoe rental. Two men were drunk
and came to Osterlind and rented a canoe from him. The canoe capsized – one of the men
was hanging on to the canoe for over half an hour before drowning; the other survived
 Held: Osterlind did not have a duty to Hill
o Osterlind had no duty to rescue the Hill
o Osterlind’s actions were nonfeasance
o Exceptions to the rule
 In certain circumstances, if the defendant had some hand in placing the plaintiff at risk of harm,
the defendant may incur a duty to take reasonable steps to warn, protect, or rescue.
 A voluntary undertaking to warn, protect, or rescue can generate a duty to take care to warn,
protect or rescue.
 Certain special relationships generate affirmative duties of care.
 See Baker; Restatement 2nd §314A (mentioned in Baker v Fenneman).
o (1) A common carrier is under a duty to its passengers to take reasonable action:
 (a) to protect them against unreasonable risk of physical harm, and
 (b) to give them first aid after it knows or has reason to know that they
are ill or injured, and to care for them until they can be cared for by
others.
o (2) An innkeeper is under a similar duty to his guests. 
o (3) A possessor of land who holds it open to the public is under a similar duty to
members of the public who enter in response to his invitation.
o (4) One who is required by law to take or who voluntarily takes the custody of
another under circumstances such as to deprive the other of his normal
opportunities for protection is under a similar duty to the other.
o Affirmative duties to rescue and protect
 Generally, NO affirmative duty to rescue
 Misfeasance- YES duty
o If you’ve done something to injure them in the first place, you have a duty to
rescue/mitigate
 Nonfeasance- generally NO duty to rescue
o Exceptions
 special relationships (camp counselor, teacher, psychiatrist)
 manipulate by arguing that the relationship is special, policy
reasons for special duty (Tarasoff)
 joint venture  duty to rescue (line between observer and participant)
(Theobald)
 social host liability
 general rule: no duty
 exceptions
o bars – YES duty
o no serving to minors
o flagrant cases - “knew or reasonably should have known
that intoxicated guest would present risk of injury”
 Baker v Fenneman & Brown Properties

8
 Facts: This was the case where the customer went to a Taco Bell where he collapsed. The
cashier rushed over to him; he got up and said he was okay. Plaintiff fell over again,
seriously injuring himself on the counter. Plaintiff brought an argument stating that the
lack of action by the defendant cause his injury
 Held: a possessor of land, if he knows or should know that an invitee on the premises
needs assistance, that possessor must take steps to assist, even if the possessor did not
cause the need
o businesses have a duty to assist customers injured on their premises
 Invitees – injuries and assistance is a cost of the business
o Restatement Second § 314, Duty to Act for Protection of Others
 The fact that the actor realizes or should realize that action on his part is
necessary for another’s aid or protection does not of itself impose upon
him a duty to take such action.
o To Taco Bell’s claim that they are not a hospital, and should not be held to the
standards of a hospital, the Court said that they do not have to do anything
extraordinary; merely call an ambulance. There is no extra cost to training or
equipment needed
 Also, there is an incentive for business not to appear callous
 This is an exception to OSTERLIND; this is not a general duty

(b) Premises Liability


o Premises liability: conditions on land, not activities
 Claims for carelessness in maintaining unsafe premises that cause physical harms brought against
property owners and occupiers (“possessors”) of the premises have traditionally been treated as a
distinct class for purposes of duty analysis.
 Under traditional common law doctrine, whether a duty is owed, and what sort of duty it is,
depends on the status of the plaintiff vis-à-vis the property. In particular, the duty owed turns on
whether she is an invitee, licensee, or trespasser.
 Today about 1/2 of the states have abandoned the licensee/invitee distinction. Less than 1/5 have
abandoned the status-based framework altogether.
 These courts apply a single reasonable person standard of liability
o CA and NY are included in this group
o Categories of entrants: trespasser, licensee, invitee
  Definition Duty Exceptions
Trespasse One who:  General rule:  3 main exceptions
r  intentionally landowner owes no duty to a  Constant trespass
and without consent or trespasser to make her land safe, to on a limited area
privilege enters or warn of dangers on it, to avoid o If the
remains on landowner’s carrying on dangerous activities on owner has reason to know
property it, or to protect the trespasser in that a limited portion of her
 for own any other way land is frequently used by
purpose, pleasure or  But, must not trespassers, she must use
convenience wilfully, wantonly or reasonable care to make the
recklessly harm premises safe or at least
warn of dangers
 Discovered

9
trespassers
o Once the
owner has knowledge that a
particular person is
trespassing, the owner is the
under a duty to exercise a
reasonable care for the
trespasser's safety
 Children

Licensee One who is:  General rule: the  


 granted owner does not owe a licensee any
permission or consent duty to inspect for unknown
(express or implied) of the dangers
landowner to enter the  If the owner
land, knows of a dangerous
 but who does condition, she must warn the
so for his own purpose licensee of that danger
rather than any purpose of
the landowner
 The main
class of people who
classify as licensees are
social guests
Invitee Someone who:  General rule: The  
 enters by owner does owe an invitee a duty
invitation (express or of reasonable inspection to find
implied); hidden dangers
 enters in  Also, the
connection with the owner must use reasonable
landowner’s business or care to take affirmative
with an activity the action to remedy a
landowner conducts or dangerous condition
permits to be conducted  Duty of due case: the
on his land; owner owes an invitee the duty of
 and offers reasonable care. In particular:
mutuality of benefit or  Duty to
benefit to the landowner inspect: the owner has a duty
 also applies to inspect her premises for
to land held open to the hidden dangers. O must use
public at large reasonable care in doing this
  inspecting
 Warning: the
giving of a warning will
often, but not always,
suffice. If O should realize
that a warning will not
remove the danger, then the
condition must actually be
remedied
 Control over

10
third persons: reasonable
care by O may require that
she exercise control over
third persons on her
premises

o Leffler v Sharp
 Facts: This was the case about the man who entered the bar. He went to the bathroom, where he
saw people smoking on the roof; kind of a balcony area. This area could only be accessed by a
small window about three feet off the ground; the door to the area was locked and the glass was
stenciled “Not an Exit”. He fell through the roof and suffered injuries
 Held: Leffler was an invitee when he entered the Quarter Inn; he became a trespasser when he
entered the roof. When he became a trespasser, the duty to maintain safe premises no longer
applied; when he became a trespasser, the possessor only owed a duty not to cause wanton or
willful injury on him
 Mississippi's 3 step process for determining premises liability
o Classifying the status of the injured person as an invitee, licensee, or trespasser;
o Determining the duty owed to the injured party;
o Asking whether duty was breached by a landowner or business operator.
 Leffler on statuses for premises liability
o An invitee is a person who goes upon the premises of another in answer to the express or
implied invitation of the owner of occupant for their mutual advantage.
o A licensee is one who enters upon the property of another for his own convenience,
pleasure, or benefit pursuant to the license or implied permission of the owner.
o A trespasser is one who enters upon another’s premises without license, invitation, or
other right.
o A trespasser enters another’s property “merely for his own purposes, pleasure, or
convenience, or out of curiosity, and without any enticement, allurement,
inducement or express or implied assurance of safety from the owner or person in
charge.”

(c) Pure Economic Loss


o The problem generally: Suppose that D behaves negligently towards X, in a way that causes X personal
injury or property damage. Suppose further that D’s conduct also injures P, but P’s only loss is economic,
not personal injury or property damage.
 May P recover in tort from D?
 Generally, no
o Aikens v Debow (W. Va. 2000)
 The case is important b/c It reiterates the dominant rule
 If all you can claim as a P is pure economic loss, then you’re in trouble
o You will have to point to a special relationship to recover
 There was no special relationship in this case, but the case does offer
examples of special relationships that do owe a duty
 Facts: This is the case about the truck driver that accidentally damaged a part of the highway. The
resulting damage to the highway made the plaintiff’s motel much more difficult to access. There
was no personal injury or even property damage
11
 Held: The court ultimately held that there was no duty by Debow to Aikens, because there was no
physical type of injury – nothing to which the claim could be parasitic
 The court may have been concerned that permitting this claim could have opened a flood
of litigation – all businesses affected would be able to bring a claim
 MAJORITY RULE: there is no duty to avoid causing pure economic loss in the absence of a
special relationship
 MINORITY RULE: a heightened foreseeability standard; defendant owes a duty to avoid causing
pure economic loss to persons of a foreseeable certain class
o Some relationships that have been recognized as permitting a pure economic loss suit
 Relationship between accountants and not just their clients, but those who loan money to, or
invest in, their clients.
 Relationship between lawyers and non-clients harmed by negligent will-drafting.
 Courts differ in how broadly or narrowly they extend these duties to non-clients.
o New Jersey= minority view
 People express case
 Discrete group of Ps the harm was foreseeable

B. The Breach Element


(i) Duty, Breach, and the Meaning of “Negligence”
 BREACH is the element of fault. It has two meanings – the INJURY, or the BREACH OF DUTY. The breach of duty is
the failure to conduct oneself with ordinary or reasonable care, or ORDINARY PRUDENCE. This is typically a jury
issue, unlike duty, as it is highly fact dependent.
o The GENERAL RULE is that a person owes an ordinary degree of care, the degree of care that an ordinarily
careful person would take to avoid causing harm. There are some exceptions (engaging in dangerous
activities like blasting, keeping and training wild animals) that result in the application of a strict liability
standard, instead of negligence. Under strict liability, even if a defendant acts with reasonable care, he is
still liable.
 Reasonable person standard
o Ordinary negligence
o Professional negligence
 Professionals have a high standard of care, higher than the ordinarily prudent person.
 Extraordinary standard of care
 Alternatives
o Rules of law
o Violation of statute
 Problems of proof: Res ipsa loquitur

(a) Ordinary Care


 Myers v Heritage Enters
o Duty to take ordinary care to avoid injuring another
o Facts: This is the case of the registered nurses who misused the Hoyer lift, resulting in the injury of
the decedent
o The plaintiff argued that the nurses should have used a professional standard of care, whereas the
defendant argued for the general rule

12
 The court agreed with defendant, stating that the defendants did not receive enough training
to be considered expert professionals, nor was the contraption something that an ordinarily
prudent person would know how to use
o Elements:
 Injury: yes, broken legs
 Duty: yes, the injury to the decedent was reasonably foreseeable
 Breach: NO; there was no special duty of care because the technicians are not specially
trained, and the contraption is so odd looking
o Jury instructions on negligence and breach
 When I use the term ‘negligence’ in these instructions, I mean the failure to do something
which a reasonably careful person would do, or the doing of something which a reasonably
careful person would not do, under circumstances similar to those shown by the evidence.
The law does not say how a reasonably careful person would act under those circumstances.
That is for you to decide.
 See IPI Civil (2000) No. 10.01
 It was the duty of the defendant’s employees, before and at the time of the occurrence, to use
ordinary care for the safety of Mary Prillmayer. This means it was the duty of the defendant’s
employees to be free from negligence.
 See IPI Civil (2000) No. 10.04
 Martin v Evans
o Facts: This is the case of the truck driver who backed up and into the defendant. There were
conflicting accounts by witnesses – one said that the plaintiff was leaning on the car between the
vehicles, the other stated that he moved between the vehicles
o Elements:
 Injury: yes, Martin suffered injuries to the arm and leg
 Duty: yes, general duty of care
 Breach: unclear – a jury question to ascertain the facts (i.e. the credibility of the witnesses
o PH/Held: The trial court had overridden the jury’s credibility determination, holding Evans guilty of
negligence
 Court holds that this was wrong, and it is a matter of jury determination

(b) Extraordinary Standard of Care


 if D has a higher degree of knowledge, skill, or experience that the “reasonable person,” D must use the
higher level
 Jones v Port Authority of Allegheny Co.
o Duty to take extraordinary care to avoid injury another
o Common carriers owe the highest duty of care to passengers
o Facts: This is the case of the bus that pulled out and stopped suddenly without the doors closing
properly resulting in the injury of the plaintiffs
o Held: The court held that bus was a “common carrier”, which owes the “highest duty of care” to its
passengers – remands new trial for jury to reconsider with the professional standard of care
 A common carrier is a commercial enterprise that holds itself out to the public as offering to
transport freight or passengers for a fee

(c) Strict Liability


 It does not matter how careful the D is If duty was violated, D is liable
13
o No inquiry into D’s prudence under strict liability standards
 Negligence standard vs. strict liability standard
o Negligence: even if defendant injures plaintiff, defendant is not subject to liability of defendant used
ordinary care
o Strict liability: if defendant injures plaintiff, defendant is liable
 Pingaro v Rossi
o Strict liability: duty to avoid injuring another
 NJ imposes strict liability in these situations- regardless of reasonable care
o Facts: This is the case of the gas company meter checker coming onto a property and being attacked
by the possessor’s dog
o Elements that plaintiff must prove:
 Injury: yes, dog bite
 Duty: yes, plaintiff is a licensee, on property by right. Defendant has more than a simple
warranty to warn. This is helped because defendant does not dispute that plaintiff had a right
to be on the property

(d) Negligence
 Campbell v Kovich
o Negligence is conduct that falls below the expected standard of care that a reasonably prudent person
would have exercised under like circumstances.
 D wasn’t negligent because the garden boy exercised adequate care in mowing the lawn (he
inspected the path, removed debris, etc)
o Facts: This is the case of the thing that came flying out when the guy was mowing his lawn –
Campbell accusing Kovich of breach of duty
o Held: court adopted the ORDINARILY PRUDENT PERSON STANDARD – no reasonable jury or person
could conclude that this person did not act with ordinary prudence
 There were questions of fact:
 It is not clear if the offending piece of rubber came from the mower; however, this is
irrelevant – if the person surveyed the lawn before him, he would have fulfilled his
duty of ordinary care
 Therefore, no material fact issue for jury contemplation
 Kovich could have been more careful, but what he was doing did not require a special duty of
care beyond the general rule
 Adams v Bullock
o A party is not negligent where he has taken reasonable precautions to avoid foreseeable dangers.
 D was not negligent (D was adhering to the relevant statutes/regulations, D took reasonable
precautions)
 Not unreasonable to fail to take care against extraordinary scenarios.
 If a defendant is on notice of a prior incident of the sort that injured plaintiff, its failure to
guard against such an incident is more likely to be found to show a lack of ordinary prudence.
o Conforming with custom provides a reason to find ordinary care
o Facts: This was the case of the young boy who was walking along a bridge, swinging a piece of wire
that was 8 feet long. On the side of the bridge, about 4-5 feet below the top of the parapet, there was a
trolley wire. The wire struck the trolley wire, electrocuting the boy
o Held: The Court determined:

14
 No reasonable person could find the defendant as negligent:
 The use of the overhead wire was lawful
 The wires were not placed somewhere that is easily reachable or accessible to a
passerby
 No special danger at the place informed the defendant of a special need for
precaution here rather than anywhere else, and no such accident had occurred here
before
 The cost to the defendant of insulating all wires would be astronomical

(ii) Defining the Person of Ordinary Prudence


 Reasonable person standard (objective test): how would a reasonable person have acted under the
circumstances?
 Rule and exceptions re the “ordinary person”
o Rule: The fault standard asks whether the defendant behaved as would an ordinarily-constituted person
acting reasonably under the circumstances. The standard is not adjusted downward for attributes that
lessen a particular defendant’s capacity to be careful, nor is it adjusted upward to reflect a given
defendant’s heightened capacity to be careful.
o Vaughan v Menlove
 Ordinary prudence is the applicable standard
 Just because Menlove is an idiot does not relieve him of the breach of duty
 Leading the case for the idea that the negligence standard is an objective one (Determined by
what a reasonable person would have done)
 Facts: This case was about the rick that caught on fire. Defendant was warned about the hazard
that his rick posed to cause the kind of damage that occurred. He responded with “I’ll risk it” –
assumed the risk
 Held: The Court rejected the defendant’s claim that he acted as prudently as possible, because he
clearly did not
o Exceptions and qualifications:
 1. Courts adjust the standard for certain defendants’ physical disabilities;
 2. Courts only rarely adjust it for mental disabilities;
 3. A child is generally judged as compared to an average child of same age/experience (unless
engaged in an adult activity);
 4. In many jurisdictions, no duty for kids of “tender years” (e.g. <7);
 Tender years doctrine
o a child can avoid a claim of negligence if they are not a person of reasonable age,
intelligence, and experience under the circumstances to know how to act
properly. The assumption is that young children cannot recognize or appreciate
risk. Notably, this only applies to children, not the mentally handicapped,

15
perhaps because children are less capable of doing serious harm to another
person.
o Bringing a claim against a parent would require the plaintiff to have special facts
to show that the parents should be aware of the child’s misbehavior, and
therefore a duty on their part to properly supervise the child.
 Appelhans v McFall
o Kids under 7 in Illinois have no duty (the kid issue was not about breach; the
parent issue is)
o Holding: court applies existing tender years doctrine – can’t be liability here
 The Court thinks that the Tender Years Doctrine is somewhat arbitrary –
a child does not automatically know what ordinary behavior at a certain
age is
 However, the Court considers that below the age of 7, a child
cannot be held to the same standard of duty
 Between ages 7-14, a court must consider that a child would
have to act as prudently as the ordinary child of like age
 Beyond the age of 14, the Doctrine does not offer protection
 But court does introduce an age-related reasonable person standard
for consideration – says if child is engaging in adult activities, he
could be held to the reasonable adult standard
 Objective standard kind of like imposing strict liability on people
– want to discourage children from engaging in adult activities,
so impose an objective standard in those instances
 The Court also states that the plaintiff failed to establish that the parents
knew or should have known about their child’s behavior, and therefore
did not establish that there was negligence in supervision
o Facts: This was the case about the plaintiff, who was 66, walking on a road when
a five-year-old ran into her from behind on a bicycle, causing her to fall and
break her hip
o P’s Arg: Appelhans claims that the parents failed to properly instruct their son on
the use of his bike, and that they knew or should have known that his youth
would prevent him from acting responsibly around others
 5. Courts sometimes adjust for special competence of certain professionals.
 Qualities of a reasonable person
o What a reasonable person knows
o How the reasonable person responds to emergencies
o Does the reasonable person follow customary practice?
o The physical attributes of the reasonable person
o Mental incapacity or mental illness
o What standard will children be held to?

(iii) Industry and Professional Custom


 Professionals are “required to exercise the skill and knowledge normally possessed by members of that
profession or trade in good standing in similar circumstances.”
 Relies heavily on custom, especially in medical negligence

16
o Compliance with industry and professional standards of care can be probative of ordinary care, but not
dispositive. There are frequently overlaps between the two, but when the custom care is not equally
covered, then the ordinary care standard is applied.

(a) Custom/ The TJ Hooper Rule


o Custom: courts generally allow evidence as to custom for the purpose of showing presence or absence of
reasonable care. However, the evidence is generally not conclusive
 Evidence by D: thus, where D shows that everyone else in the industry does things the way D did
them, the jury is still free to conclude that the industry custom is unreasonably dangerous and
thus negligent
 The TJ Hooper
o The fact that most tugboats in the industry do not yet have radios does not
prevent the jury from holding that D’s lack of radio was negligence
 even if defendant has acted in a customarily prudent manner, that alone
is not enough to establish that the defendant took the necessary level of
reasonable care under the ordinary person standard
o Custom= relevant and probative, but not dispositive
o Facts: This was the case of the tugboats that were out at sea tugging the barges
full of coal – one of them sank. There was a storm brewing in the Atlantic; there
had been radio broadcasts updating ships in the area of the status of the storm.
Neither of the two tugboats had a proper radio – the presence of radios was, at
the time, not a customary standard
o Held: The Court held that just because it was not an industry standard does not
mean that it is not something that an ordinary person would utilize
 The anti-Hooper rule: for certain professions (highly technical) the court will look to the
respective standard practice
o Medicine: prudent patient standard, not prudent doctor
 Proof by P: conversely, proof offered by P that others in D’s industry followed a certain
precaution that D did not, will be suggestive but not conclusive evidence that D was negligent
o Exceptions to the TJ Hooper Rule
 The TJ HOOPER RULE does not apply in cases of malpractice (if a professional acted in a manner
typical of members of a field, there is no breach when the defendant followed professional
conduct that ultimately harmed the plaintiff);
 In a malpractice case, a plaintiff must be able to provide expert testimony to show that
the defendant did not follow such standard professional behavior
 The INFORMED CONSENT RULE – when it comes to malpractices cases, what a reasonable patient
may expect may be used in lieu of the professional standard

(b) Medical Malpractice


o GENERALLY: professionals must act with the level of skill and learning commonly possessed by
members of the profession in good standing
 Good results not guaranteed just that the professional will use the requisite minimum skill and
competence
o Under JOHNSON and CONDRA, if a plaintiff can establish that a professional followed industry or
professional standards, then it is dispositive over the ordinarily prudent person standard
o Johnson v Riverdale Anesthesia Associates
17
 standard of care in medical malpractice is that which is employed by the profession generally –
not what an individual physician would do – so question isn’t relevant
 Anti-TJ Hooper rule  custom is determinative in the case of medical malpractice
 Don't have to allow cross examination
 Not relevant to either of the plaintiff's two potential purposes of the questioning
o Current standard of duty in the field and credibility of defendant's expert
o Irrelevant because about medical profession generally, not what one individual
would do
o OVERRULED BY LEGISLATION
 Facts: This was the case about the decedent who, during surgery, suffered from a severe adverse
reaction to an anesthesia she received during surgery. It was alleged that Riverdale failed to
“preoxygenate” decedent, which is a procedure where the patient is given a measure of pure
oxygen as a reserve should oxygen supply be disrupted during surgery
 Held: the testimony of how one expert would act is insufficient to establish if a breach existed
because a single doctor’s actions do not reflect industry standards
 In order to show medical malpractice, a plaintiff must be able to prove that the doctor
violated the degree of care and skill required by a physician; such a standard of care is
that typically employed by medical professionals
o Condra v Atlanta Orthopedic Group
 A court may not exclude evidence of an expert witness’s own practices insofar as they relate to
the applicable standard of care.
 Condra holding on the expert questioning issue
o We hold that evidence regarding an expert witness’ personal practices, unless
subject to exclusion on other evidentiary grounds, is admissible both as
substantive evidence and to impeach the expert’s
 Facts: This was the case about the patient who was misdiagnosed, and the doctor’s treatment
regimen caused her to suffer greater injury. Plaintiff brought a claim that the defendant failed to
properly monitor her blood count during therapy sessions. Defendant says that blood
measurements are not a standard of care in the treatment, and that they would not have been able
to catch her disease
 Held: It was error for the trial court to exclude evidence of defense experts’ personal medical
practices.
 The Court overruled the Johnson rule – GA narrows the ability to bring expert testimony
o It is not just the substance of the expert opinion, but also the credibility
o The GA Statute stated that an expert witness must have actual knowledge and
experience of at least 3-5 years in a field
 Post-Johnson Rule Johnson Rule doesn’t apply
o Largey v Rothman
 Facts: This was the case where the doctor failed to warn of specific risks regarding a procedure of
t(x). The gynecologist found a vague mass in Largey’s right breast, and part of the treatment
required a biopsy of two lymph nodes; she was not informed of the biopsy. The biopsy resulted in
the swelling in her right arm and hand
 PH: The jury had been instructed to consider if a doctor provided information that showed that
the doctor acted in a manner that a medical practitioner would use
 Held: The Court remands for a new trial stating that the wrong standard was used – it is not what
care a reasonable practitioner would use, but what a reasonable patient would expect
18
 the new governing standard is the prudent patient standard  because:
o Desirable scope of disclosure varies from patient to patient
o Extra-medical factors must go into consideration
o Professional standard isn’t consistent with self-determination
o Plaintiffs may have difficulty finding an expert to testify against the doctor –
community “wall of silence”
 About ½ of states have moved to the prudent patient standard over
professional standard

(c) Informed consent


o Part of a physicians duty is to adequately disclose the risks of the proposed t(x) to the patient in advance.
 The rule requiring adequate disclosure is called the rule of “informed consent”
 The doctor must disclose to the pt. all risks associated with the t(x) which are sufficiently
material that a reasonable pt. would take them into account in deciding whther to undergo
the t(x)
o Failure to get the patient’s adequate consent is deemed a form of malpractice and
thus a form of negligence
o Lack of informed consent cause of action
 Informed consent standards
 “Professional standard:” Doctor must disclose the risks that a reasonable doctor in the
community, of like training, would customarily make in similar circumstances.
 Minority version: Doctor must disclose the risks that a reasonable medical practitioner
would make under similar circumstances.
 “Patient standard:” Doctor must disclose “material” risks. A risk would be deemed
“material” when a reasonable patient, in what the doctor knows or should know to be the
patient’s position, would be “likely to attach significance to the risk or cluster of risks” in
deciding whether to forego the proposed therapy or to submit to it.
 NY Court of Appeals on informed consent
 Orphan v. Pilnik (2010):
o Facts: P claimed she was not informed by D that the procedure would result in a
6.5 cm scar, she did not give informed consent to remove benign breast lump
o Held: To succeed in a medical malpractice cause of action premised on lack of
informed consent, a plaintiff must demonstrate that
 (1) the practitioner failed to disclose the risks, benefits and alternatives to
the procedure or treatment that a reasonable practitioner would have
disclosed and
 (2) a reasonable person in the plaintiff's position, fully informed, would
have elected not to undergo the procedure or treatment.

(iv) Reasonableness, Balancing, and Cost-Benefit Analysis


 In determining whether the risk of harm from D’s conduct was so great as to be “unreasonable,” courts use a
balancing test: “Where an act is one which a reasonable person would recognize as involving a risk of harm to
another, the risk is unreasonable and the act is negligent if the risk is of such magnitude as to outweigh what the
law regards as the utility of the act or of the particular manner in which it is done”
o Negligence exists if D undertakes UNREASONABLY risky conduct under the circumstances
 Not just if the risk is associated with the conduct
19
o Negligence looks at if D acted like a reasonably prudent person under circumstances NOT if D acted in
good faith, doing the best he could in his judgement

(a) The Hand Formula


o US v Carroll Towing Co (Risk-Utility Balancing)
 Facts: This case was about the barges that broke free and smashed into the tanker, which caused
one of the barges to sink. The barge was carrying grain
 Held: all three plaintiffs were equally responsible for the damages
 The court decided that there were three variables to look at:
o The probability that the barge would break away
o The gravity of the resulting injury
o The burden of adequate precautions
 The Hand Formula is mostly useful for separating out the elements of probability and loss
 The duty to provide against harm is a function of:
 B = the burden of adequate precautions to eliminate or reduce the harm
 P = the probability/foreseeability of harm occurring
 L = the gravity/severity of the resulting injury.
 Balancing equation:
 B<PL=D is negligent
 B>PL= D is NOT negligent
 Can’t just look at damages to plug into factors because never talking about one case, Instead:
using the perspective of the actor pre-accident, across all cases meeting certain criteria, what is
the range of harms that could happen? What is the range of probability and magnitude of loss?
What is the burden? (includes opportunity costs, unintended consequences (could make cleaning
product less caustic but reduces ability to clean, medication could get rid of one side effect but is
part of burden that could introduce another side effect or be less effective) so don’t just think of it
in terms of dollars)
 Advantages of Hand formula
o Provides a method of analysis – draws out certain variables for court to look at
o Promotes efficient behavior
 Also promotes societal maximization
 Criticisms of Hand formula
o Assumes all costs can be monetized – some costs aren’t easily monetizable
 i.e. cost to bargee of essentially keeping him as a prisoner on board
o Overlooks distributional considerations – might result in systematic under-protection of some
groups that we are concerned about (i.e. children)
o Could be seen as failing the cost-benefit test that it sets out
 May be seen as imposing other kinds of administrative costs – requires courts to have a
lot of information, inappropriate application could generate error costs
o Rhode Island Hosp Trust Nat’l Bank v Zapata Corp
 Facts: Zapata’s employee was issuing fraudulent checks under the company’s account. Zapata
didn’t realize the foged checks, despite the fact that it received the account statement from it’s
bank. A couple months later, Zapata realized that its bank had been issuing payment for the
forged checks and it brought suit against the bank to recover the amount of the forged check
 Issue: employee forged checks, Zapata didn’t notice until a few months had passed

20
 Zapata says bank didn’t exercise due care in its forgery detection system
 Holding: “an industry-wide practice that saves money without significantly increasing the
number of forged checks that the bank erroneously pays is a practice that reflects at least
ordinary care.
 Illustrates use of the Hand formula  must look at what additional costs and
benefits would flow from adding an additional precaution
o Zapata says bank should check every individual check – adding this precaution
would cost bank an additional $125K
o Benefits of adding this additional precaution wouldn’t have increased detection
of forged checks
o So PxL = 0 – burden of precaution clearly larger than expected benefits

(v) Proving Breach: Res Ipsa Loquitur


 Proof of negligence:
o (1) direct, (2) circumstantial, (3) Res Ipsa Loquitur
 Res Ipsa Loquitur (“the thing speaks for itself”):
o Allows P to point to the fact of the accident, and to create an inference that, evne without a precise
showing of how D behaved, D was probably negligent
 Example: Byne v Boadie
 The case that first applied Res Ipsa
 Facts: A barrel of flour falls on P’s head as he walks below a window on the street. At
trial, P shows that the barrel fell out of a window of D’s shop, and that barrels do not fall
out of windows without some negligence
 Held: By use of the res ipsa doctrine, P has presented enough evidence to justify a verdict
for him, so unless D comes up with rebuttal evidence that the barrel did not come from
his shop or was not dropped by negligence, D will lose
o The effect of res ipsa
 Permits an inference that D was negligent, even though there is no direct evidence of negligence
 When res ipsa is used, P has met his burden of production, and is thus entitled to go to the jury
 When Res Ipsa comes up
o P will bring the case, P offers evidence of the fact that D owes P duty, offer evidence about causation,
damages, BUT P can’t offer evidence about what D did that breached the duty of reasonable care
 The 3 elements (Kamabt v St. Francis Hosp. requirements for res ipsa instruction)
o (1) the event must be of a kind that ordinarily does not occur in the absence of someone’s negligence;
 P must demonstrate that the harm that occurred does not normally occur except through the
negligence of someone
o (2) Negligence most likely by D (it must be caused by an agency or instrumentality within the exclusive
control of the defendant);
 P must demonstrate that the negligence was probably that of the D
 Not always required
o The 3d Restatement dropped this requirement
 Multiple D’s
o If there are 2+ Ds, and P can show that at least one of the Ds was in control,
some cases allow P to recover
 Especially where all of the Ds participate together in an integrated relationship
21
o (3) it must not have been due to any voluntary action or contribution on the part of the plaintiff.
 Plaintiff was a passive victim
 P must establish that the accident was probably not due to his own conduct
o Other requirements
 No direct evidence of D’s conduct in connection with the event
 Evidence more available to D
 Some courts also require that evidence of what really happened to be more available to D
than to P
o Facts: This case was about the surgical pads there were found inside of a person who after surgery –
resulted in fatal post-op infection. The New York court did not give a res ipsa instruction. Medical
malpractice is not a common place for res ipsa because accidents are not uncommon, and people are
generally careful. However, an 18-by-18-inch pad is not ordinarily discovered inside a patient’s abdomen
after a surgery absent negligence
o D’s Arg: the plaintiff was not a passive victim and had swallowed the pads
o Held: this is not a case requiring expert testimony, but rather something for a jury to decide and is capable
of contemplating
 If you can satisfy all 3 elements…
o Helps you get to jury on breach and causation, when you otherwise might not be able to get to jury
 Contexts where res ipsa will arise
o Airplane accidents
o Single Car accidents
 Rebuttal evidence
o General evidence of due care
 If D’s rebuttal is merely in the form of evidence showing that he was careful never enough to
give D a directed verdict
o Rebuttal of res ipsa requirements
 If D’s evidence directly disproves one of the requirements D will get a directed verdict

C. The Causation Element


(i) Key Terms and Concepts
 2 components of causation
o Actual cause (also sometimes called “cause-in-fact” or “factual cause”):
 Was the victim’s injury brought about by (among other things) the defendant’s carelessness?
 Inquiry is usually guided by the “but-for” test (also known as the “sine qua non” or
“counterfactual” test),
 i.e.: Would the plaintiff have been injured even if the defendant had acted with the care,
he was duty-bound to exercise?
 But for D’s negligence, the injury would not have occurred
o Proximate cause (sometimes called “legal cause”):
 Did the actor’s carelessness bring about the victim’s injury in such a fortuitous manner that it is
inappropriate to hold the actor responsible for the injury?
 Restatement (Third) on Factual Cause:
o §26 Factual Cause

22
Tortious conduct must be a factual cause of harm for liability to be imposed. Conduct is a factual
cause of harm when the harm would not have occurred absent the conduct. Tortious conduct may
also be a factual cause of harm under §27.
o §27 Multiple Sufficient Causes
 If multiple acts occur, each of which under §26 alone would have been a factual cause of the
physical harm at the same time in the absence of the other act(s), each is regarded as a factual
cause of the harm.

(ii) Proving Actual Causation Under the Preponderance Standard


 Preponderance standard
o This standard requires the jury to return a judgment in favor of the plaintiff if the plaintiff is able to show
that a particular fact or event was more likely than not to have occurred.
 Some scholars define the preponderance of the evidence standard as requiring a finding that at
least 51% of the evidence favors the plaintiff’s outcome.
o You can submit circumstantial evidence, but can’t ask jury just to guess
 Reasonable inference is okay; speculative conjecture is not
 But for test
o Muckler v. Buchl
 Facts: a woman who had lived in her apartment building for 7 years fell down and broke her hip,
and died four months later. The staircase was dark – a witness stated that they could not see
where they were going without holding onto a railing
 But for the poor lighting, would Muckler have fallen down the stairs?
 Needs to be probable for the jury to decide
 P needs to show that the light factor was more probable than not to have caused the
injury- essentially, dismissing or outweighing other factors, such as drunkenness, age,
illness, etc.
 Premises liability revisited.
o Butts v.Weisz
 Facts: the elderly couple who visited their friends for the first time – licensees, due the care of
warning. They had gone on a cursory tour of the home, because they wanted to get dinner as they
were hungry. Mrs. Weisz saw Mr. Butts head the wrong way to the bathroom but did not correct
him. Mr. Butts fell down the stairs and died. The stairway dark and there was a step
 Held: if Weisz’ had warned Butts, Butts probably would not have fallen
 The injury was identified as Butts’ fall from the landing
o Speculative testimony from the expert – the only thing they have to go by is the
manner in which Butts fell is consistent with someone who missed the step
o Just because conditions make something more likely to happen does not
necessarily mean the cause
o “Consistency with the fall” is insufficient
 The preponderance standard – was it probable that the defendant’s actions caused Butts
to fall?
o They were required to warn guests of any hidden traps – but as the Butts had
passed over that step several times, it seems like they may have been reasonably
informed of the danger
 No causal link, no sufficient/substantial evidence to raise genuine issue of material fact with
respect to causation, could have fallen for other reasons.
23
 Restatement 2nd Section 342
 A possessor of land is subject to liability for physical harm caused to licensees by a
condition on the land if, but only if,
o (a) the possessor knows or has reason to know of the condition and should realize
that it involves an unreasonable risk of harm to such licensees and should expect
that they will not discover or realize the danger, and
o (b) he fails to exercise reasonable care to make the condition safe, or to warn the
licensees of the condition and risk involved, and
o (c) the licensees do not know or have reason to know of the condition and the
risk involved.
o Jones v. LA Fitness Int’l, LLC
 Facts: the guy who was playing basketball – there were unpadded walls on the basketball court.
 Invitee – under common law, owed a duty to maintain safe premises, and the maximum
kind of duty.
o If he were a licensee, he would have a weaker claim, because defendant only has
a duty to warn of hidden dangers, and this was not a hidden danger
 Evidence does not need to prove but for cause, just reasonable basis that D’s negligence was
more likely than not the cause. Court applies the substantial factor test (increased risk does not
equal cause)
 P just needs to provide evidence that provides a reasonable basis
 P has met this standard
 Pennsylvania duty to business invitees
 A possessor of land is subject to liability for physical harm caused to his invitees by a
condition on the land if, but only if, he
o (a)  knows or by the exercise of reasonable care would discover the condition,
and should realize that it involves an unreasonable risk of harm to such invitees,
and
o (b)  should expect that they will not discover or realize the danger, or will fail to
protect themselves against it, and
o (c)  fails to exercise reasonable care to protect them against the danger.
 A possessor of land is not liable to an invitee for injuries caused by a condition on the
land whose danger is known or obvious to the invitee, unless the possessor should
anticipate the harm despite such knowledge or obviousness.
 In this case, the expert testimony was permitted, as he is an expert in safety standards – it is not
“strictly necessary” but the testimony might assist a jury in determining whether LA Fitness court
conformed to a reasonable standard of care
 This is reiterating the Daubert standard under Rule 702 of the Federal Rules of Evidence
 Expert must have some genuine and specialized knowledge
 Proposed testimony must be scientifically valid or reliable
 Expert’s testimony must be able to assist the jurors
 On actual causation – court frames it as “would Jones still have been injured if there were more
padding, more space, etc. (conformed to industry standards)” for jury to contemplate

(iii) Multiple Necessary and Multiple Sufficient Causes


 Summary of the standards

24
o Multiple necessary causes (MacDonald)
 When each of two or more careless acts functions as a necessary condition of an injury, each is an
actual cause of that injury.
 Need to show particular D in question was required to get the result
o Multiple sufficient causes (Ford Motor Co v Boomer)
 When each of two (or more) independent careless acts committed by different actors would by
itself have generated an injury that happens to have occurred when both acts were performed, and
when neither act can be said to have brought about the injury prior to the other act’s having
brought about the injury, each careless act is deemed to be an actual cause of the injury. This is so
even though neither act satisfies the but-for test. In this situation (sometimes called “concurrent
causes”) courts tend simply to suspend but-for.
 Need to show that the risk that D exposed the P was enough to cause the injury
 Each individual D must be able to be sufficient
 exposure to the defendant’s product alone must have been sufficient to have caused the harm
o NESS (Necessary Element of a Sufficient Set)
 "if there is a set of conditions that together are sufficient to bring about a result, then each of the
conditions necessary to the sufficiency of that set can be said to be a cause of the result."
 Wannal v Honeywell International
o To overcome summary judgment in a negligence action involving multiple causes, a plaintiff’s expert
must sufficiently provide medical or scientific evidence directly linking the defendant’s product to the
plaintiff’s injury.
 This case suggests boomer is helpful in one sense, but in other ways it is tough on Ps need
impossible expert evidence
 Depends on scientific evidence, and exposure needing to pass a specific standard
o Wannall’s reliance on Boomer
 As the Virginia Supreme Court stated, a defendant in such a case may only be found liable “if the
jurors, after hearing the testimony and evidence, believe that a negligent exposure was more
likely than not sufficient to have triggered the harm.” Boomer.
 Thus, it is clear that, at a minimum, in order to survive summary judgment, a plaintiff must come
forth with some medical or scientific evidence that exposure to the defendant’s product was
sufficient, in and of itself, to trigger the injury.
 Boomer stated . . . that “experts must opine as to what level of exposure is sufficient to cause
mesothelioma, and whether the levels of exposure at issue in this case were sufficient.”

(a) multiple necessary causes


 Example- A and B, who each weighs 175 lbs., lean against the car simultaneously. The car rolls over the cliff
and is destroyed. Against whom can O establish a claim of negligence?
o Both
 MacDonald
o Facts: two cars collided, and one of them struck McDonald, knocking her down and causing her to be
dragged under until it came to a stop
o PH: Plaintiff sued both drivers, alleging that both were equally negligent
o Held: The court found the defendants to be JOINTLY AND SEVERALLY LIABLE
 The defendants’ actions individually were not THE cause of her injuries
 The defendants’ actions individually were each AN ACTUAL cause of her injuries

25
o The Iowa court states that joint liability does not require coordinated action if the careless acts act as
the function of a single injury

(b) multiple sufficient causes


 Example- A and B each weigh 310 lbs., and they lean against the car simultaneously. The car rolls over. Car
needs 300 pounds to roll over
o Multiple sufficient- Boomer
 Any 1 of them could've tipped the car
 Can go after A and B
 Ford Motor Co v Boomer
o Boomer’s preferred standard
 The standard that, in this case, exposure to the defendant’s product alone must have been
sufficient to have caused the harm is both an accurate articulation of our concurring cause law
and perfectly plain to the average juror. This standard constitutes the cause-in-fact portion of
the proximate cause requirement in concurring cause cases.
o “Sufficient to have caused the injury” becomes the test
 Conduct of defendant was sufficient to have caused the injury if evidence is sufficient to have
caused the injury – the conduct may nor or was not necessary
 A sufficient cause – if two or more individuals act, both are careless and committed by
different actors, and any act alone would have caused the injury, if either act caused the
injury before the other act, both have caused the injury, neither are necessary, both are
sufficient
o Facts: Lokey, who served as a Virginia State Trooper for 30 years – among his duties were to perform
mechanical checks on vehicles. He would blow on break debris to clear away dirt. He remembered
breathing in visible dust in the garages. He was not provided protective clothing or masks. The brakes
in the cares used 50% asbestos material, and likely held 100% of the market for the vehicles used by
the state troopers at the time. Expert testimony stated that that kind of asbestos can cause cancer.
o PH: Expert for the defense stated that asbestos-containing brakes cause no higher chances of lung
cancer than to those who do not spend time around such brakes. Noted a higher incidence of cancer
for those who worked in shipyards, which Lokey had worked in during his youth. Admits that every
exposure to asbestos is a substantial contributing factor to the development of cancer

(c) NESS
 Example-A, B, and C, who each weigh 175 lb, lean against the car.  It takes 300lb to tip it over.
o MacDonald nor Boomer help
 Any 2 would get car to move, but don't know which 2
 Furthermore, any 1 is not sufficient
 No good with Boomer or McDonald
o So, none of them is necessary (any 2 of them would tip it over). None of them is sufficient (175 is
smaller than 300).
 But if a court applied NESS it could find A, B, and C all to be causes.
 look for sets of conditions that are sufficient to bring about the result in question.
o Sets of conditions that are sufficient to equal 300 are the following: A & B;
A & C; B & C; A & B & C.

26
 Now we're looking for conditions that are necessary to the
sufficiency of the set that they are in. In other words, people whose
weight is required to make a particular set add up to 300. 
 As to the first combo, both A & B are required, because if
we took either away, we wouldn't be at 300;
 with the second combo, both A & C are required;
 with the third, both B & C are required.
 With the final one, none of them is required, because you
don't need all 3 to get to 300; so, we disregard that set
 So, A, B, and C are all causes under the NESS test because each of them is a necessary
member of at least one sufficient set. (In fact, each of them is a necessary member of 2
sufficient sets.)

(iv) Causation and Tortfeasor Identification


(a) Alternative causation
 An exception to actual cause allows the P to shift the burden of proving causation onto multiple Ds, even
though only one of them caused the injury
 General rule: if more than 1 person is an actual and proximate cause of P’s harm, and the harm is indivisible,
each D is liable for the entire harm
o Most usually kept at 2 Ds
o Allows the P to shift the burden of proving causation onto multiple Ds, even though only one of them
cause the injury
 Each D is liable for the whole damage whether they are deemed to be acting in concert or
independently
 The Elements in Action:
o All D before Court
o P cannot prove which one did it
o Alleged negligence of 2 Ds was exactly the same
 Rationale for the rule
o Ds know better than P how P got injured
o Fairness and good sense
o We have causation to at least 1 of the defendants
o Although only 1 may have shot, both are culpable because they both shot
 Summers v Tice
o Facts: This was the case where the guys were hunting and two of them shot at the third one, who
suffered an injury
o Issue: The problem was that it could not be determined which one had injured Summer
o Held: Court rejects the probability standard and adopts the ALTERNATIVE LIABILITY DOCTRINE
 Both defendants held prima facie liable even though each was just as likely as the other to
have caused the injury
 Defendants can come forward with affirmative defenses
 The evidentiary deadlock can be broken by proving that one or the other was the cause
o The Elements in Action:
 Each of Two Defendants Acted in an Identical Manner: Yes
 Carelessness of One but Not the Other Was Actual Cause: Yes
27
 No Suggestion that Any Other Tortfeasor Was Involved: Yes
 Plaintiff Cannot Prove Which Defendant Caused Injury: Yes

(b) Market share liability


 If, through no fault of her own, a P is unable to identify which manufacturer injured her and she sues a group
of manufacturers, who together form a substantial % of the relevant market. Each D is subject to liability on
its market share %, unless it can disprove that it did not cause the P’s injury.
o A shift of burden to D to prove that they did not cause the injury
o Each D is subject to liability unless it can disprove that its drug caused P’s injury
o For any D that cannot disprove, P can recover % of damages based on D’s market share, as remains
after the removal of certain Ds
 Adopted in Sindell v. Abbott Labs
o Facts: Sindell brought an action against 11 drug companies on behalf of herself and women similarly
situated. They have suffered from vaginal and cervical cancer growths
o Held: The court rejected the plaintiff’s argument of CONCERT OF ACTION LIABILITY – a form of joint
liability that originates from explicitly or totally agreed upon conduct between parties
 Court rejects on the grounds that no evidence supports coordination, or even a conspiracy
 Court adopts MARKET SHARE LIABILITY
 A shift of burden to the defendant to prove that they did not cause injury
 Each defendant is subject to liability unless it can disprove that its drug caused
plaintiff’s injury
 For any defendant that cannot disprove, plaintiff can recover percentage of damage
based on defendant’s market share, as remains after the removal of certain defendants
 other possible theories the plaintiff could have used
 Alternative liability
 Acting in concert
 “enterprise”/ “industry-wide” liability
 NY has endorsed market share liability with a twist
o Irrebuttable presumption of causation
 Each D is liable to each P for its national market share

D. Aligning the Elements: Proximate Cause and Palsgraf


(i) Proximate Cause
 Proximate cause brings up the Q of whether there is something about the actual causal connection between D’s
actions and P’s injury that warrants freeing D from liability for that injury notwithstanding the connection.
 A D’s actions must be BOTH the actual and proximate cause of P’s injury
 General rule: The D is liable for all harmful results that are the normal consequences and foreseeable scope of
increased risk of his acts
 Proximate cause tests:
o Natural and ordinary
o Directness
o Foreseeability
o The risk rule/ “scope of the risk” test

28
(a) Natural and ordinary test (no longer used)
 was the P’s injury the natural and ordinary consequence of the D’s tortious conduct?
 Replaced by directness
 Ryan
o Facts: 19th century rail yard – sparks from the engines ignited a shed located in the yard, which then
spread to the plaintiff’s home 150 feet from the shed. The house, along with several others, were
destroyed in the fire
o Held: The Court held for the defendant, stating that it was not “natural and ordinary” for a fire to
spread beyond the first structure that ignites (the One Leap Rule)
 Though it does not make sense, it was probably made out of a concern for public policy
 Plaintiff loses because of a finding that it is not natural or ordinary for fire to spread from its
source to another structure AND THEN to spread further (here, from engine to shed to
plaintiff’s house).
 Union pump
o Facts: the pump that caught fire at a gas station. It had caught on fire twice before. This time, it
caused a fire; the firefighters asked employees of the store to help them put out the fire in a certain
area. The manager took Allbritton to the area over a pipe rack several feet off the ground, because he
often took that route. It was covered and slippery in firefighting foam. They got across the first time
okay, Allbritton fell the second time. Allbritton sued the manufacturer for damages, alleging that but-
for the pump catching fire, she would not have fallen and hurt herself
o PH: The Trial Court granted summary judgment for the defendant; it was a but-for cause, but not a
proximate one.
o Held: Upheld by the Supreme Court
 It was determined that the plaintiff’s injury was too attenuated
 It was a substantial factor, but a mere condition to the injury
 The consequences of the defendant’s actions (the fire) had “come to rest” at the time
of the plaintiff’s injury
 If the D merely brought about a condition that made P’s injury possible, not the same as
causing injury. Was not foreseeable.
 Majority says no proximate cause using a very grab-bag approach
 Remoteness – says too remote a connection
 Policy considerations
 Substantial factor test
 Attenuation – says plaintiff’s injuries too attenuated
o Concurrence clearer- applies foreseeability test

(b) Directness Test (no longer used)


 Applied when injury follows closely to the breach, in which injury and breach are not separated by significant
time or space
o A P’s injury must be temporally and spatially linked directly to D’s breach
 See Polemis (Kings Bench 1921)
o Facts: the steamship that caught on fire. The ship carried flammable benzene; the falling of a plank
followed by a rush of flames caused by the fumes from the benzene resulted in the destruction of the
ship
o Held: directness of the worker’s fault in dropping the plank and the injury is close enough

29
 It may seem attenuated, but in terms of directness, it was not remote – it was an instantaneous
injury resulting from an act
 Causation element satisfied where workers carelessly knocked a wooden plank off the deck
of a ship into the hold, as a result of which a spark ignited flammable vapors. Careless acts
found to have been direct cause of explosion; lack of foreseeability of explosion found
“irrelevant
 Replaced by foreseeability

(c) Foreseeability Test (Current Test)


 An act is the proximate cause of the injury, if and only if it was reasonably foreseeable to cause the injury
when the D acted. An actor ought only be responsible for harm that reasonably foreseeably results from his
actions
 See Wagon Mound I (Privy Council 1961)
o Established foreseeability as legal test for proximate cause
o Facts: the ship, Wagon Mound, whose crew carelessly released a large slick of furnace oil into
Sydney Harbor. A nearby ship was being repaired with welding torches; the supervisor learned of the
oil spill ceased activities, then ordered them to resume after discussing with the Wagon Mound wharf
owner. The next day, several pieces of debris caught on fire, resulting in the destruction of many
docks
 D’s ship spills oil into a bay. Some of the oil adheres to P’s wharf. The oil is then set afire by
some molten metal dropped by P’s workers, which ignites a cotton rag floating on the water.
P’s whole dock then burns
o Held: D is not liable, because the burning of P’s dock was not foreseeable consequence of D’s oil
spill, and thus the oil spill was not the proximate cause of the damage. This is true even though the
burning may have been the “direct” result of D’s negligence
o
 the proximate cause as to the fire was a direct consequence of defendant’s actions
 This new standard overruled POLEMIS – the actor ought only be responsible for
damages that reasonably appear to result from his actions
 The most common proximate cause standard
 Jolley v. Sutton London Borough Council (H.L. 2000)
o An act is the proximate cause if and only if it was reasonably foreseeable to cause the injury when
defendant took action. An actor ought only be responsible for harm that reasonably foreseeably
results from his actions.
o Facts: two teenagers were fixing up a boat that had been left in a common area for months . The two
boys had been fixing up the boat in the hopes of sailing it in search of buried pirate treasure. The
wrench had collapsed and paralyzed one of the boys
o Issue: what needs to be foreseen? The exact nature of plaintiff’s actions or the type of plaintiff’s
actions?
o Held: the boat was an attractive nuisance
 While the defendant argued that this was not “normal child’s play” (i.e. not exactly
foreseeable) the plaintiff argued that the use of the boat by the boat was no different in type
(they are of a similar kind) – one judge calls it a “genus” of harm – not the exact
circumstances, but a category of harm
 The Court stated that this kind of injury could not have happened to mere children

30
(d) Scope of Risk Test
 Limits liability to physical harms that result from the risks that made the actor’s conduct tortious
 A breach of a duty of reasonable care that actually causes an injury is a proximate cause of that injury only if
the injury is a realization of one of the risks that rendered the defendant’s conduct careless.
 See Third Restatement §29:
o “An actor’s liability is limited to those harms that result from the risks that made the actor’s conduct
tortious.”
o Third restatement goes with scope of risk not foreseeability
 Overlaps with foreseeability- closely related
o Often the result is the same
o Even when court's say they're applying foreseeability analysis, they're often looking at scope of the
risk
 Ventricelli
o Facts: This is the case about the guy who rented a car with a defective latch. He pulled over, popped
his trunk open, and was struck by another car
o Held: The court held that this was too attenuated The court held that the scope of risk rule only
applies to certain unsafe conditions that are foreseeable
 If the trunk flew open to obstruct his view, or something else that would fall within “certain
scenarios

(ii) The Relational Aspect of Breach of Duty: Palsgraf


 The general rule that D is liable only for foreseeable consequences is also usually applied to the “unforeseeable P”
problem.
o That is, if D’s conduct is negligent as to X (in the sense that it imposes an unreasonable risk of harm upon
X), but not negligent as to P (i.e., does not impose an unreasonable risk of harm upon P), P will not be
able to recover if through some fluke he is injured
 Duty is owed to particular Ps, particularly those in the zone of danger, who is put at risk (within
the range of apprehension)
 No duty to unforeseeable Ps
 Palsgraf v Long Island RR Co established the relationality component between duty and breach
o Facts: X, trying to board D’s train, is pushed by D’s employee. X drops a package, which (unknown to
anybody) contains fireworks, which explode when they fall. The shock of the explosion makes some
scales at the other end of the platform fall down, hitting P
o Held: P may not recover against D. D’s employee may have been negligent toward X (by pushing him),
but the employee’s conduct did not involve any foreseeable risk of harm to P, who was standing far away.
Since D’s conduct did not involve an unreasonable risk of harm to P, and the damage to her was not
foreseeable, the fact that the conduct was unjustifiably risky to X is irrelevant. D’s conduct was not the
“proximate cause” of the harm to P
o Cardozo: (Majority)
 There was no duty to someone standing on the other end of the platform
 There was a duty to those in “the orbit of the danger” that was reasonably foreseeable
 The RELATIONAL ASPECT – the railroad was not careless towards her because she was
outside of the scope of risk
 There was no breach, as there was nothing in the situation to suggest that the package could be
fireworks

31
 The jury had determined that there was a breach – this is Cardozo overturning the
“reasonable” jury
 There was no proximate cause of injury – it was unforeseeable that the small, unassuming
package could create an explosion that would loosen the shingles and harm the plaintiff
 Cardozo followed a Two-Step Analysis:
 A jury cannot find that defendant was careless with reference to the plaintiff
o The guard’s actions could be wrong in relation to the package owner or people
nearby, but not someone who was standing far away
 A negligence claim cannot piggyback on defendant’s carelessness towards another person
o The plaintiff sues in his or her own right for a personal wrong, but not as a
vicarious beneficiary of the breach of duty to another
o Andrews: (Dissent)
 Andrews rejected the idea of Relationality, and viewed tort law differently from Cardozo:
 There was a breach – a person owes a duty to avoid causing anyone injury
 He discussed a much larger scope, arguing that the issue was not just an affront to the plaintiff,
but an antisocial conduct that was eligible for legal sanctions
 Everyone is responsible for their actions to others – when injuries result from unlawful acts, we
are liable for the consequences, in so far as the act can be considered a proximate cause of the
harm

(iii) Superseding Cause and Affirmative Duties


 Intervening cause: a force that takes effect after D’s negligence, and which contributes to the negligence in
producing P’s injury
o an unforeseeable intervening cause will absolve the first tortfeasor of liability
o Superseding cause: subset of intervening cause that frees the D from liability (because there was no
proximate cause) (not helpful because it is just label and doesn’t give us content to make a decision)
 These causes supersede or cancel D’s liability
 The Third Restatement Rejects Superseding Causes, and relies instead on Apportionment – a Third Party that is
more culpable than the defendant will pay a larger percentage of damages; this avoids the all-or-nothing rule of
Superseding Cause
 Pollard and Clark
o In Clarke, the boy had collected residual explosive powder; despite his parents’ directions, he continued
to collect it and it exploded, injuring Pollard.
 The various intervening activity was undertaken independently by the teenager were enough to
break the causal chain
o In Pollard, the teenager disposed of the canister of glycerine. When it exploded on someone else, the
court found that McDowell’s actions did not break the causal chain
 Port Authority of NY & NJ v. Arcadian Corp. (3d Cir. 1999)
o A common carrier is a commercial enterprise that holds itself out to the public as offering to transport
freight or passengers for a fee
o Facts: This is the case of the bus that pulled out and stopped suddenly without the doors closing properly
resulting in the injury of the plaintiffs
o Held: the bus was a “common carrier”, which owes the “highest duty of care” to its passengers – remands
new trial for jury to reconsider with the professional standard of care

32
o A manufacturer of a component that is adulterated for criminal purposes is not liable for injuries caused
by the adulterated end product where the component itself was not defective or unreasonably dangerous
and where the component’s adulteration was not reasonably foreseeable.
o Rulings against plaintiff in Arcadian
 As regards negligence claim [and products liability claim]:
 Defendants had no duty to plaintiff (under either NY or NJ law)
 Even if a duty and breach, no proximate cause (under either NY or NJ law)
 Also, as regards failure to warn claim:
 No duty
 No showing of causation]
 Fast Eddie's v Hall
o Facts: the woman who was drunk at a bar; the bar (Fast Eddie’s) asked two patrons to take her outside.
One patron took her back to his home and left her in his car. The other patron came to his home, found
her in the car, sexually assaulted and murdered her
o Held: The murder was not foreseeable; thus, Fast Eddie’s not liable
 Claim on negligence and failure to protect the victim – nonfeasance
 No duty was owed to the decedent after she had left the premises
 Claim on negligence per se in the violation of the Dram Act – misfeasance
 Even if there was negligence per se, the breach was not a proximate cause of the murder
o The nature of the murderer’s personality and state of mind was the proximate
cause, not sending her home with someone who ultimately did not directly harm
her
o Murderer’s intentional acts were the intervening cause
 Also, the murder was arguably unforeseeable
o Fast Eddie's on Proximate and Superseding Cause
 A party’s act is the proximate cause of an injury if it is the natural and probable consequence of
the act and should have been reasonably foreseen and anticipated in light of the circumstances.
 However, a willful, malicious criminal act of a third party is an intervening act which breaks the
causal chain between the alleged negligence and the resulting harm.

E. Statutory Supplements
(i) Negligence Per Se
 Negligence per se- a statute establishes a duty, which is allegedly breached by the D
o unexcused violation of a statute is a BREACH as a matter of law
o Way of establishing breach, without having to persuade the court that a duty of ordinary care was violated
o This does not mean that you are liable still have to prove other elements (causation, duty, damages)
o A vast majority of states follow this approach
o The key issue is under what circumstances do we use the statute in the tort system because talking about
statutes designed for purposes other than tort
 Steps for determining negligence per se
(1) Does the statute or regulation set a standard of conduct? (Duty)
(2) Did defendant violate the standard without an excuse? (Breach)
(3) Was the violation an actual cause of plaintiff’s injury? (Actual Cause)
(4) Was the plaintiff a member of a protected class? (Duty)
 Bayne v Todd Shipyards Co
33
 Not enough to point to a relevant statute must show that the statute is there to protect
people like you from things that happened to you
 Facts: the delivery man who was unloading a truck when he fell off of the loading
platform. The federal statute requires there to be a guard rail. D argued that the plaintiff
was not an employee, hence no statutory duty to the deliveryman, and no breach of
statute
 Held: just because someone is not an employee, the statutory protections still extended
 Per the restatement, a reasonable person should following an administrative regulation
whose purpose is:
o To protect a CLASS OF PERSONS which includes the one whose interest is
invaded;
o To protect the particular interest which is invaded;
o To protect the interest against the kind of harm which has resulted;
o To protect that interest against the particular harm from which harm results.
(5) Was the incident the kind intended the statute seeks to prevent? (Proximate cause)
 Victor v Hedges
 Facts: the car was parked on a sidewalk near an apartment building. D and P were
standing next to the car when another car drover over the curb and injured the P. P claims
that if the defendant had not violated statute by parking on the sidewalk, she would not
have been struck. But for negligence, she would not have been harmed
 Held: The injury was not the type the statute tries to protect against
 Restatement (2nd) on negligence per se
o The court may adopt as the standard of conduct of a reasonable man the requirements of a legislative
enactment or an administrative regulation whose purpose is found to be exclusively or in part
 A reasonable person should follow an administrative regulation whose purpose is:
 (a) to protect a class of persons which includes the one whose interest is invaded, and
 (b)  to protect the particular interest which is invaded, and
 (c)  to protect that interest against the kind of harm which has resulted, and
 (d) to protect that interest against the particular hazard from which the harm results.
 Restatement (3rd) on negligence per se
o An actor is [careless] if, without excuse, the actor violates a statute that is designed to protect against the
type of accident the actor’s conduct causes, an
 Dalal v. City of New York (N.Y. App. Div. 1999)
o Example of negligence per se
 A restriction placed on the license requiring the use of glasses created a duty that relates directly
with the operation of the vehicle
o Facts: This was the case about the automobile that vehicle that crashed into another one because
defendant was not wearing her eyeglasses. Defendant claims that she was able to see without her glasses,
and was not wearing them during the accident
o Held: it was a mistake to not to tell the jury about negligence per se. jury must be told if there was a
breach

F. Defenses
(i) Contributory Negligence and Comparative Responsibility
 States have moved away from contributory negligence to comparative fault
34
o Contributory negligence- all or nothing for P
o Comparative fault- parse out various contributors
 Contributory negligence:
o Typically a complete defense.
o Doctrine of “last clear chance” developed to protect plaintiffs in some cases
 Comparative fault/ comparative responsibility:
o Typically not a complete defense, but leads to reduction in P’s damages.
o But in modified comparative responsibility regimes (adopted by about 2/3 of the states), P can lose out
entirely if found to bear responsibility at or over a certain percentage (varies by state).

(a) Contributory Negligence


 Contributory negligence is an affirmative defense, to which if a D is able to prove that the P was himself
careless, and the carelessness of the D was not a cause of the injury.
o Completely bars recovery
 There are some exceptions
o Does not apply when dealing with intentional torts;
o Jury is capable of nullification;
o The Last Chance Doctrine – if the decedent had a last chance at avoiding an accident, then he does
not enjoy the protections of contributory negligence.
 Typically, a complete defense.
 From around 1970 to 1990 replaced in all but 5 states by comparative fault

(b) Comparative Fault/ Comparative Responsibility


 most states have adopted comparative fault standard
o under this standard, some at-fault Ps can recover, but the compensatory damages are awarded to those
who recover are reduced in proportion to fault
 Typically, not a complete defense, but leads to reduction in P’s damages.
 But in modified comparative responsibility regimes (adopted by about 2/3 of the states)
o P can lose out entirely if found to bear responsibility at or over a certain percentage (varies by state).
 United States v. Reliable Transfer Co. (U.S. 1975)
o Facts: a ship that ran aground on a sandbar because of a lack of a flashing light at a lighthouse – suing
the Coast Guard. The ship captain had a chart, searchlight, telephone, lookout, etc., but relied on his
own guesswork in turning a loop. Found to be a 25% fault by Coast Guard, 75% fault by plaintiff
o Held: when two or more parties are at fault to cause property damage, then each is to pay for the
damages in proportion to their fault
 If they are equally at fault, or the fault cannot be determined/split, then are to be held equally
at fault
 Damages sustained by Reliable should be subject to the doctrine of comparative fault, not the
divided-damages rule.
 Hunt v. Ohio Dept. of Rehabilitation & Correction (Ohio Ct. Cl. 1997)
o Facts: the prisoner who was operating a snowblower and injured her hand. She had reached into the
snowblower, and had not been instructed not to. Her instruction on how to use the snowblower was
lacking, though the officer who gave the instruction did so to the best of his ability
o Held: Unusual: judge comes up with calculation (usually jury decides)

35
 40% is the amount by which we should reduce P’s damages because her negligence
constituted 40% of the cause of her injuries

(c) Divided Damages


 Not what you typically have in torts
 Split evenly between the amount of parties involved
o Ex: if 2 parties50/50

(ii) Assumption of Risk


 A Plaintiff is said to have assumed the risk of certain harm if she has voluntarily consented to take her
chances that the harm will occur. Where such an assumption is shown, the P is, at common law, completely
barred from recovery
o Express: written agreement or waiver in which the P agrees to give up some or all of her potential
claims against D, even if there might have been negligent on D’s fault
 If P explicitly agrees with D, in advance of any harm, that P will not hold D liable for certain
harm, P is said to have expressly assumed the risk of that harm
o Implied: nothing written or stated, voluntarily took it on
 Even if P never makes an actual agreement with D whereby P assumes the risk, P may be
held to have assumed certain risks by her conduct. Here the assumption of risk is implied
 Distinguished from contributory negligence
o Often P’s assumption of risk will also constitute contributory negligence
 Reasonable assumption of risk:
 But this is not always true: sometimes conduct that constitutes assumption of risk is
not contributory negligence
 Defense to reckless conduct:
 Distinguishing between assumption of risk and contributory negligence may be
important where D’s conduct was reckless: contributory negligence is not a defense
to reckless conduct, but assumption of the risk generally is

(a) Express assumption of risk


 This involves an express agreement by the plaintiff to waive warranty against the defendant for
defendant’s wrongdoing.
o This is ordinarily a complete bar to recovery.
o Even when a waiver is contractually sound and covers the nature of the injury, a court may
choose to strike it down for policy reasons.
 Jones v Dressel
o Facts: This was the case about the young man who went skydiving, and then the plane crashed;
Jones survived, and brought a suit against the company. Jones had signed a waiver, but this was
when he was a minor. Jones had the option of paying more and surrendering the waiver –
Adhesion Contract
o Issue: whether the waiver was valid, as Jones was a minor, and whether the waiver even covered
the scope of the injury that was suffered
o Held: the contract was fairly entered, and the contract clearly stated a waiver provision  no
adhesion K
 Factors that the court must consider in deciding if an exculpatory agreement is valid
 (1) the existence of a duty to the public
36
o Indicia of invalidity: Indicia of invalidity: a business of a type suitable
for public regulation; a service of great importance to the public –
often a necessity for some; seller holds itself out as willing to perform
this service for anyone (eligible) who seeks it; as a result of essential
nature of the service, seller has decisive bargaining advantage;
standardized adhesion contract, with no way to buy back one’s
protection; person or property of purchaser is placed under the control
of seller, subject to risk of carelessness by seller.
 (2) the nature of the service performed
 (3) whether the K was fairly entered into; and
 (4) whether the intention of the parties is expressed in clear and unambiguous
language
 He met 3 and 4 but not 1 and 2?
o Furthermore, there is not a duty to the public due to the public – this was not a “common carrier,”
the use of the plane was merely incidental to the nature of the business. This is not something that
a lot of people engage in.
 3 Exceptions to express assumption of risk
o (1) when the party protected by the clause (typically the D) either intentionally causes the harm,
or else brings about the harm by acting in a reckless or grossly negligent way
o (2) when the bargaining power of the party protected by the clause is grossly greater than that of
the other party, typically a status the court finds to exist only when the good or service being
offered is “essential”
o (3) where the court concludes that there is some overriding public interest which demands that the
court refuse to enforce the exculpatory clause
 Dalury v S-K-I LTD
 Facts: This is the case where plaintiff collided with a metal pole that formed a
part of the control maze for the ski lift line. Dalury had signed a waiver, but had
to sign the waiver to ski
 Held: The court determined that the waiver was not valid by applying the Tunkel
factors:
o The skiing industry is incredibly important to Vermont’s economy and
requires regulation;
o The public frequently comes on to the premises; furthermore, the
defendant would frequently advertise to entice locals to engage in skiing;
o The plaintiff was not able to ski unless he signed the waiver. There was
not an equitable balance of power between them;
 More eclectic and specialized activities have more interest in upholding waivers
because participants are presumed to possess a greater knowledge of the risk
 restatement 2d on express assumption of risk: section § 496B, comment b
o Exculpatory agreements “will generally be upheld” where they are
“freely and fairly made, between parties who are in an equal bargaining
position, and there is no social interest with which they interfere.”

(b) Implied assumption of risk


 Implied assumptions of risk involve people who proceed voluntarily to encounter known risks.

37
o As such, IAoRs do not involve contracts.
o These IAoRs typically involve recreational activities, and mere awareness of a background risk
does not entail plaintiff to have assumed the risk – a defendant must show that a plaintiff went to
meet the risk, despite the knowing the likelihood of the risk occurring.
o Remember – implied assumption of risk is different from contributory fault. A person’s actions
may not be negligent, and they go willingly towards the danger.
o New York has a “hybrid” approach – namely, plaintiff’s damages will be diminished in
proportion to their ‘culpable conduct’ attributable to plaintiff. The assumption of risk counts to
the culpable conduct.
 Thus, assumption of risk turns into a partial defense without turning it entirely into
comparative fault. (IAoR is a complete bar to recovery if plaintiff is injured by
carelessness while participating in a sport or in some recreational way).
 Harder to figure out when you have a fact pattern that may involve an implied assumption of risk
 2 requirements
o For D to establish implied assumption, he must show that P’s actions demonstrated that she: (1)
knew of the risk in Q, and (2) voluntarily consented to bear that risk herself
 Knowledge of risk
 The risk must be one which was actually known to P, not one which ought to
have been known to her
 Voluntary assumption
 There is no assumption of the risk, if D’s conduct left P with no reasonable
choice but to encounter a known danger (duress)
 How the Virgin Islands deals with implied assumption of risk
o Smollett v Skayting Dev Corp
 Facts: P had had some experience skating in the past. She went to a skating fundraiser –
the area did not have a guard rail, and she asked about it, and was informed that it was the
trend. The area was not overly crowded, and there were safety people to make sure
people didn’t get hurt. Plaintiff rolled off of the elevated rink and injured herself
 Held: she was an experienced skater who made the decision to skate regardless of her
awareness of the risk – the fact that she was aware of the risk was what was a major
factor
 In the Virgin Islands, for a D to win claim of implied assumption of risk they must
 show (1) P knew the risk, (2) P made a voluntary free choice to do the risky
activity, (3) P’s behavior was reasonable
o If the behavior is unreasonable, deal with it as an issue of comparative
fault
 Court says (1) yes, (2) yes, (3) yes  so D won
o (3) reasonable because she’s experienced, spoke with the workers
 Dissent
 “Defendant can sustain its burden by proving that plaintiff knew of the risk,
appreciated its character, and voluntarily chose to accept it.” [quoting 3rd Circuit
precedent]
 Key takeaways
 Evidence favors verdict that she did assume the risk finds for D despite jury
verdict for P

38
 Comparative fault
o Once contributory negligence shifts to comparative fault, states have a
choice on whether to apply implied assumption
 Some states choose not to enforce implied assumption; rather,
they apply comparative fault
 The Virgin Islands hold onto implied assumption, but only when she made a
reasonable choice
o Unreasonable choice comparative fault

(iii) Immunities and Exemptions from Liability


 Prevents you from suing
 All about identity of D and the relationship to P
 Much less immunity now, but there are still some pockets left

(a) Intra-familial and charitable immunities


o Historically, spouses could not sue spouses, parents could not sue children vice versa
o Could not sue charity or charitable institutions
o Blanket immunities faded away in almost every state in the 1970s
 Got rid of spousal immunity
 Most states say parents aren’t immune from suits from their children
o Why the fading away?
 Shifting of gender norms, slimming down of the patriarchy
 No institution should be above the law
 Many more individuals and organizations are insured can guard against the suit

(b) Governmental/ Sovereign immunity


o Sovereign immunity: Covers situations where government entities cannot be sued
o Governmental immunities & exemptions
 Federal:
o Common law sovereign immunity significantly reduced by the Federal Tort Claims Act
(FTCA), which holds
federal government liable for certain torts its agents commit in the course of their
employment.
 There are exceptions, where immunity remains
 “discretionary function” exemption
o No liability may be based upon the government’s exercise of a
discretionary or policy-making f(x), even if the discretion is broad
 Riley v United States
o Facts: MAILBOXES. P sues the U.S. under FTCA, alleging USPS
negligence in the placing and maintenance of, and failure to relocate,
mailboxes. P’s view of traffic was obscured by mailboxes; he pulled
onto
the highway thinking the road was clear, and was hit by a truck. No
federal statute or rule mandated the USPS to locate the mailboxes at
any particular place.

39
o Held: P’s claim dismissed because the alleged negligence
constituted a discretionary act immune from a lawsuit
o FTCA provision on discretionary function
 The FTCA does not waive immunity for “the exercise or
performance or the failure to exercise or perform a
discretionary function or duty on the part of a federal agency
or an employee of the Government, whether or not the
discretion involved be abused.” 28 U.S.C. §2680(a).
o Test for when FTCA’s discretionary function exception applies
 1. The conduct at issue must be discretionary, involving “an
element of judgment or choice.”
 2. “The judgment at issue be of the kind that the
discretionary function exception was designed to shield.”
 i.e. must be a judgment “grounded in social,
economic, and political policy.”
 Both prongs satisfied: (1) USPS made a discretionary choice,
(2) policy decision FTCA intended to exempt from liability
o The federal government is subject to liability “if a private person would be liable to the
claimant in accordance with the law of the place where the act or omission occurred.”
o Generally, FTCA doesn’t cover intentional wrongdoings (but some exceptions for law
enforcement).
o Westfall Act exempts federal employees from being held individually liable for torts
committed in the scope of
their employment. (Note that there’s a carveout for alleged constitutional violations).
 State:
o Common law sovereign immunity significantly reduced by state counterparts to the FTCA.
o Many states have equivalents of the Westfall Act.
 Local/municipal/city:
o Not “sovereigns,” but common law courts tended to exempt them from liability for
governmental activities.
o The “public duty” rule can negate the duty element in negligence claims against local
government. See Riss.
o (And see its extension to quasi-governmental service providers in Belle Realty).

(c) Liability Exemptions: No Duty Rules for Local Government & Private Entities
1. Local Government
 Got rid of spousal immunity
 Riss v City of New York
 Facts: P sues NY after being injured by her ex-bf, Pugach. She claims NYPD was
negligent when they failed to protect her from her ex despite being on notice of threats
from him.
 Held: court dismisses the case. Doesn’t want to interfere with the city’s allocation of
responsibility- legislative, not judicial, determination
o Public duty rule invoked  although gov owes certain duties to the public at-
large, it doesn’t owe those duties to any individual members of the public

40
 Court recognizes that plaintiff was a foreseeable victim of harm by def,
but is reluctant to say there is a duty because concerned about open-
ended liabiltiy
 Riss rationales
o No affirmative duty to protect in this circumstance: this situation is
distinguishable from one where the police “undertake responsibilities to
particular members of the public and expose them, without adequate protection,
to the risks which then materialize into actual losses."
 Majority’s strongest legal point
o It is not the role of courts to determine how police resources are used (even in
cases of those seeking protection “based on specific hazards”).
o The limits of liability for the city would not be predictable [and liability would
potentially be huge].
 Takeaways
o If the cops do provide assistance, then an affirmative duty is created
o Suit against a city, so the sovereign immunity doctrine doesn’t apply
o The court blocks the claim
 Does so for the same reason the immunities exist
 Court getting into arenas it doesn’t have the power to get into
o Public duty doctrine
 The rule that a governmental entity (such as a state or municipality)
cannot be held liable for an individual plaintiff's injury resulting from a
governmental officer's or employee's breach of a duty owed to the
general public rather than to the individual plaintiff.
o Externality
 A consequence or side effect of one’s economic activity, causing another
to benefit without paying or to suffer without compensation

2. Private Entities
 Strauss v Belle Realty Co
 Facts: CON EDISON ELECTRICITY OUTAGE. P sues her tenant and con Edison for
electricity outage. Alleged negligence in the performance of its duty to provide
electricity.
 Held: Con Edison is not liable
 Strauss rationales
o It is . . . the responsibility of courts, in fixing the orbit of duty, “to limit the legal
consequences of wrongs to a controllable degree”, and to protect against crushing
exposure to liability. “In fixing the bounds of that duty, not only logic and
science, but policy play an important role”.
o Considerations of privity are not entirely irrelevant in implementing policy.
Indeed, in determining the liability of utilities for consequential damages for
failure to provide service—a liability which could obviously be “enormous,” . . .
— courts have declined to extend the duty of care to noncustomers.
o Permitting recovery to those in plaintiff’s circumstances would . . . violate the
court’s responsibility to define an orbit of duty that places controllable limits on
liability.
41
o No fact-finding hearing needed in order to define duty: given a) obvious impact;
b) impossibility of fixing a rational boundary beyond contractual relationship;
and c) the societal consequences of rampant liability.
 Policy is the whole story
o Privity is relevant
 MacPherson don’t need to be in privity to bring a claim
 Dissent: Wants Con Edison to demonstrate that extending liability to noncustomers
would have a catastrophic result before limitation is placed on its duty
 Takeaways
o Suit against private entity (large utility)
o Not immunity because not government
o Analogous to the kinds of situations that makes governments immune
o Diversion from established standards
 Macpherson
 On policy basis of not wanting to cause too much liability
o If you can show
 (1) almost like a government in providing an essential service (2)
liability would be crushing if a tort suit is successful can get judge to
carve you out under duty

G. Damages and Apportionment


 Possible relief at trial
o Compensatory damages
 Money; jury has wide discretion in choosing the amount.
 Supposed to be fair and reasonable compensation in light of the losses that the plaintiff has suffered.
o Punitive damages
 Money; jury has wide discretion in deciding whether to award any.
 Serving to punish.
 Some jurisdictions require that plaintiffs suffer something other than nominal damages in order to be
eligible for punitive damages.
 Rarely awarded in negligence claims; found more often in battery, for example. (Standard instances
of negligence do not involve the requisite willfulness/malice/recklessness.)
o Injunctive relief
 A court order that the defendant do or stop doing something.
 Rarely found in negligence cases; more often in trespass and nuisance.

(i) Elements and Availability of Damages


(a) Compensatory Damages
o compensatory damages are awarded to Ps for their injuries suffered at the hands of the D.
 Juries are instructed to award damages that will fairly and adequately compensate the P for the
injuries sustained
o Compensatory damages fall into 2 categories
 Economic losses
 Past and future lost earnings
 Past and future costs

42
 Non-economic losses
 Victimization
 Disfigurement
 Past and future pain and suffering
o Eggshell Skull rule
 Tortfeasor takes the victim as he finds him
 Once tortfeasor commits a tort, she can’t avoid paying for some injuries that the tort lead to
 As our book puts it, “having wronged and injured another, a tortfeasor cannot be heard to
complain that the amount of damage caused to that other was much greater than anyone could
reasonably have expected because of a hidden physical vulnerability in the plaintiff.”
 This is to say that a tortfeasor is responsible for all damages caused to a victim, even if these damages
are disproportionately large, or unexpected injury stems from an unforeseen weakness of the plaintiff
o Smith v. Leech Brain & Co. Ltd. (Q.B.1962)
 Test for determining what injuries employers are responsible for, not whether they could have
reasonably foreseen the extent of the injuries, but whether they could reasonable foresee the type of
injury suffered
 Facts: steelworker who was hit in the face with a piece of molten steel. The molten steel was a
“promoting agent” that caused a cancer that then resulted in his death. A claim against the company
for negligence, presumably for maintaining unsafe premises, namely the lack of a proper guard shield
for operators. Defendant argued that they should only be held liable for foreseeable damages
 Held: the widow could recover damages for the death of decedent; D liable for the full consequences
of the damages
 The claim of proximate cause limits liability, not damages
 Foreseeability: Was burn (the injury) foreseeable?
o Then if the burn is actual cause of the cancer, then don’t require P to show that
cancer was reasonably foreseeable
 Eggshell Skull rule
o Regardless of the weakness of the decedent, the D is responsible for damages to P
that arise from D’s negligence or tortious conduct
o Kenton v. Hyatt Hotels Corp. (Mo.1985)
 Facts: This was the case about the skywalk that collapsed and killed hundreds. The plaintiff was a law
student who was severely injured, injuries including permanent spinal cord damage, spasticity of
muscles, inability to walk without muscles; she has a reduced ability to enjoy her old life or enjoy a
normal life
 Held: P awarded $800K for pain and suffering  Award not excessive
 The court split the awards in three:
o Economic Damages: $3.2 million
 Lost earnings: $2.2 million
 Medical costs: $1.0 million
o Noneconomic Damages: $0.8 million
 Pain and suffering
 Tension between: desire of jurors and sometimes jurors go crazy and we need to reign them in
 Noneconomic damages: pain and suffering
 Jury has to translate evidence of pain and suffering to dollar terms
 Here, Took the noneconomic costs to 4 million dollars

43
 Jurors have a great deal of discretion

(b) Punitive Damages


 Awarded to penalize a D whose conduct is particularly outrageous
o Negligence cases: punitive damages are awarded only where D’s conduct was “reckless” or “willful
and wanton”
 Can’t claim on merely negligent conduct
 As a P in order to get punitive damages, need to point to
o (1) some sort of physical injury,
o (2) can get compensatory damages,
o (3) some form of aggravated form of tort (willfulness, insult, oppression)
 Must meet the clear and convincing standard to convince the jury that P deserves punitive damages
 Rarely awarded
 National By-Products, Inc. v. Searcy House Moving Co. (Ark.1987)
o Facts: This was the case about the truck that smashed into a car which caused an accident, resulting in
the deaths of two. The truck driver had been driving over the speed limit; he had received six citations
for driving an overweight truck in the previous year; he probably tried to apply his brakes at the last
minute but the brakes were not working properly. He ended up smashing into the car at 70 mph . The
truck also damaged a trailer and house – the accident resulted in some property damage to Searcy .
$2,000 for compensatory damages, $100,000 to Searcy for punitive
o Held: Truck drive was not wanton, so Punitive damages not awarded
 P only sustained minor property damage, D was wanton in his actions to decedents, but not to
P as a separate claimant
o Requirements for punitive damages in this case
 An award of punitive damages is justified only where the evidence indicates that the
defendant acted wantonly in causing the injury or with such a conscious indifference to the
consequences that malice may be inferred.
 Wantonness . . . is shown by a person, when, notwithstanding his conscious and timely
knowledge of an approach to an unusual danger and of common probability of injury to
others, he proceeds into the presence of danger, with indifference to consequences and with
absence of all care....
 It is not necessary to prove that the defendant deliberately intended to injure the plaintiff. It is
enough if it is shown that, indifferent to consequences, the defendant intentionally acted in
such a way that the natural and probable consequence of his act was injury to the plaintiff.
 It must appear that the negligent party knew, or had reason to believe, that his act of
negligence was about to inflict injury, and that he continued in his course with a conscious
indifference to the consequences, from which malice may be inferred.
 Mathias v Accor Economy Lodging
o Facts: two guests stayed at a motel and were bitten by bedbugs. The D’s management knew that there
were bedbugs – an exterminator had confirmed it and offered to remove the bedbugs, to which
management and refused. The next year, bedbugs were found again in a room, and only that room
was sprayed. Clerks were instructed to call the bedbugs “ticks”. One visitor had been moved to three
rooms; after being in the first room, they had discovered bedbugs. Jury award had been $5,000 for
compensation, $186,000 for punitive. $1,000 for each room in the hotel – connecting the award to the
claim
o Held: The $186,000 in punitive damages was fair
44
 Willful/wanton standard met
 Purposes of punitive damages
 punitive damages awards were meant to deter similar action by the D in the future.
o Reported to management level and still nothing was done to fix the bed bugs
 The award was intended to punish the defendant for outrageous conduct.
o The damages were proportional to the wrongness of the D’s conduct
 Compensatory damages won’t do the job  appropriate for jury to award punitive damages

Battery, Assault, and Infliction of Emotional Distress


A. Battery and Assault
(i) Battery
(a) Battery: Prima Facie Case
 Battery: intentionally causing another to suffer a harmful or offensive touching.
o Battery requires not to have an anticipation of the harm; anticipation of the offensive touching and the
contact itself is both battery and assault
 Elements
o Harmful or offensive touching
o Intent
o P need not be aware
 Not necessary that P have actual awareness of the contact at the time it occurs
 Actor A is subject to liability to other person P for battery if:
o A acts,
o Intending to cause a contact with P;
 does not intend to be harmful or offensive; just needs to intend to do the conduct
o the contact with P that A intends is of a harmful OR offensive type; and
o A’s act causes P to suffer a contact that is harmful or offensive.

(b) Harmful or offensive touching


 harmful: causes pain or bodily damage
o Cecarelli v. Maher (Conn.Com.Pl.1943)
 Facts: P claims that D and 2 others beat him up b/c they were jealous that 3 women asked P
to drive them home.
 Held: Ps may recover under a battery tort, if there was an intentional, harmful physical attack
 offensive: damaging to a reasonable sense of dignity
o Paul v. Holbrook (Fla.App.1997)
 Facts: two co-workers – one of whom made sexual advances on the other. The defendant
ended up rubbing her shoulders. This was a sexual harassment suit brought as a battery
 Held: the D committed an offensive touching with the requisite intent
 Our court overturns the grant of summary judgment on the battery claim against
Holbrook; affirms it as regards all other claims.
o Why summary judgment for defendants on these charges?
 Holbrook: assault, intentional infliction of emotional distress,
negligent infliction of emotional distress.

45
 PMP: assault, battery, intentional infliction of emotional distress,
negligent infliction of emotional distress, negligent hiring and
retention.
 Battery definition used in Paul
 ”A battery consists of the infliction of a harmful or offensive contact upon another
with the intent to cause such contact or the apprehension that such contact is
imminent.”

(c) Intent
 Majority rule:
o It is not necessary that D desires to physically harm P. D has the necessary intent for battery if it is the
case that either:
 (1) D intended to cause a harmful or offensive bodily contact; or
 (2) D intended to cause an imminent apprehension on P’s part of a harmful or offensive
bodily contact
o Vosburg v. Putney (Wis.1891)
 Facts: the child who kicked the other child, resulting in some more serious injury, like an
infection or a broken bone. The kick was made playfully, but in a classroom outside of recess
hours
 Held: The court determined that this was a battery
 The kick was intentional and “unlawful” in the sense that it was done outside of the
framework of acceptable play
 Batteries can attach to “unlawful” behaviors, “unlawful” meaning socially
unacceptable behaviors
 3 main questions addressed in Vosburg
 Is an intent to do harm required for battery? No
o his intended conduct is unlawful and thus his intent itself is unlawful.
 Was it error to allow plaintiff’s expert witness (who was not familiar with the earlier
injury) to answer a question about what the “exciting cause” was of the inflammation
that he saw when he examined him? Yes
 Should the damages have been limited to those that the defendant “might reasonably
be supposed to have contemplated as likely to result from his kicking the plaintiff”?
no
o The wrongdoer is liable for all injuries resulting directly from the wrongful
act, whether they could or could not have been foreseen by him
o Cole v. Hibberd (OhioApp.1994)
 Facts: This was the case of the two families that were hanging out. Defendant kicked
plaintiff, breaking her tailbone. Plaintiff acknowledges that this was probably done in jest
 Held: this was a battery – it was an intentional contact intended to be offensive to the victim’s
sense of personal dignity
 Battery: yes
 Whether socially unacceptable conduct was intended, the man intended to make
contact.
o Whether this resulted in injury is irrelevant to the intent to touch
 Ohio precedent on when the assault and battery statute of limitations will apply.

46
 “Where the essential character of an alleged tort is an intentional, offensive touching,
the statute of limitations for assault and battery governs even if the touching is
pleaded as an act of negligence. To hold otherwise would defeat the assault and
battery statute of limitations.”
 Ohio caselaw on battery
 An individual is liable for battery when he or she acts intending to cause offensive or
harmful contact, and such contact results.
 “Offensive contact” is contact that would be offensive to a reasonable sense of
personal dignity.
 Minority rule:
o The intent to touch is sufficient to constitute a battery, regardless of harmfulness or social
acceptability
o Wagner v State
 Intent to make contact is all that is necessary to meet the intent element in a battery
claim
 Do not need to intend that the contact is harmful
 If there is consent under the law, there is no battery
 Second restatement §13 (adopted by Utah)
 An actor is subject to liability to another for battery if
o (a)  he acts intending to cause a harmful or offensive contact with the person
of the other or a third person, or an imminent apprehension of such a contact,
and
o (b)  a harmful contact with the person of the other directly or indirectly
results.
 Comment c. to 2nd Restatement
o If an act is done with the intention described in this Section, it is immaterial
that the actor is not inspired by any personal hostility to the other, or a desire
to injure
 Facts: the mentally handicapped man who attacked a woman in a convenience store. He was
supposed to be watched by people, but they failed to prevent the injury. Plaintiffs argue that
the state is responsible because the man did not have the capacity to be capable of intending
to do harm. Defendants argue that the man had capacity to intend contact
 Held: this was a battery – whether socially unacceptable conduct was intended, the man
intended to make contact. Whether this resulted in injury is irrelevant to the intent to touch
o Spivey
 Facts: D put his arm around P and pulled her head toward him in a “friendly, unsolicited hug”
that ultimately caused P to suffer from partial facial paralysis. P brought suit for assault and
battery and negligence. Because the suit was brought after the statute of limitations for
intentional torts had run, D argued that the acts complained of were strictly intentional and
the suit was barred.
 Held: grant of summary judgment was reversed to allow Plaintiff to proceed with her claim
for negligence.
 In conflict with Vosburg, Cole, and Paul
 For this to be a battery, require that you are intentionally trying to cause the P’s
paralysis

47
o Intent to not just cause harm, but also intent to cause specific harm

(ii) Assault
(a) Assault: Prima Facie Case
 Assault: the causing of anticipation, anxiety, or apprehension of physical harm. Assault must be a completed
tort- there is no attempt to cause assault
 Actor A is subject to liability to other person P for assault if:
o (1) A acts,
o (2) intending to cause in P the apprehension of an imminent harmful or offensive contact with P; and
 Apprehension doesn’t have to involve fear
 It is enough if P believes harmful or offensive contact was imminent
o (3) A’s act causes P reasonably to apprehend such a contact.
 In terms of apprehension, this does not mean that a plaintiff need fear the contact, in that the
contact need not be violent. A lack of fear may affect the compensatory damages
 The threat of bodily harm must be imminent

(b) Reasonable apprehension


 Beach v. Hancock (N.H. 1853)
o Even if the thing you fear never happens (or is an impossibility), if I was reasonable in thinking it
could happen, then that could mean there is a possibility of assault
o Reasonable apprehension
 Apprehension does not have to be correct just reasonable
o Facts: Fight between the two parties. D raises a gun at P and pulls the trigger twice at P (unloaded
gun but P didn’t know).
o Held: D’s conduct constituted an assault

(c) Words Alone do not suffice for a successful Assault Claim


 Brooker v. Silverthorne (S.C. 1919)
o P may have felt an apprehension, but the apprehension was not a reasonable apprehension
o In general, skeptical about allowing mere words to be the basis of a successful action
 Sometimes do allow threats to be the basis, but if this was a threat, it wasn’t sufficiently
immediate (He was not there, he was talking about an alternate reality)
o Facts: the phone operator who was threatened by the man on the other line. He threatened her by
saying “If I were there, I would break your neck”
o Held: there was no imminent danger or harm – he was not there to cause the injury, nor did he
threaten to go to plaintiff’s location
 Court also held that plaintiff’s fear of “imminent harm” was unreasonable

(d) Threats + Conduct Can be Assault; No Bodily Contact is Necessary


 Vetter v. Morgan (Kan. App. 1995)
o Whether vetter reasonably apprehended harmful conduct is a jury Q
o Definition of assault
 Assault is defined as “an intentional threat or attempt, coupled with apparent ability, to do
bodily harm to another, resulting in immediate apprehension of bodily harm. No bodily
contact is necessary.”
 Ordinarily, words alone cannot be an assault.
48
 However, words can constitute assault if “together with other acts or circumstances
they put the other in reasonable apprehension of imminent harmful or offensive
contact with his person.” Restatement (2nd) of Torts §31.
o It is not necessary that the victim be placed in apprehension of instantaneous
harm. It is sufficient if it appears there will be no significant delay. See
Restatement (2nd) of Torts §29(1), comment b (1964).
o Facts: This was the case about the woman who was driving alone on at night on an isolated road.
Morgan, driving with friends, drove up near her, screaming vile and threatening obscenities, shaking
his fist, and span on her van door
o Held: the outcome on the case hinted on Step 3 – whether Vetter reasonably apprehended harmful
conduct is a jury question

(iii) Transferred Intent


 2 types of transferred intent
o Intent transfers across victims (see White):
 D acts with the requisite intent to ward A, but happens to cause that effect on B, resulting in
liability to B
 Ex: D shoots at A, intending to cause A to believe that she is about to be shot. Bystander
B- whom D had no intention of scaring-apprehends that she is about to be shot. D is
liable to B for assault
o Intent can transfer across torts:
 So a battery D cannot defeat liability by professing that he only meant to scare the actual victim
not to actually shoot him
 Ex: if D shoots at P intending only to scare P, but manages to scare and hit P, then P can
sue for the assault and for the battery
 In re White (Bankr. E.D. Va 1982)
o Facts: D fired a gun at Tipton, an individual he previously had an altercation with. However, D missed
Tipton and the bullet struck P in the stomach. Thereafter, D filed for bankruptcy and Plaintiff filed an
action in the bankruptcy court to declare D’s debt to him nondischargeable.
o Held: because the shooting was wrongful and intentional, and because it caused Davis’s injuries, White’s
debt to Davis for those injuries is not dischargeable in White’s bankruptcy.
 A debt incurred from an action “based upon a willful and malicious” injury by the debtor to
another person” may be non-dischargeable bankrupt. Willful means deliberate or intentional
 Court says this bankruptcy statute is met
 As long as there was an attempt to injure someone and someone was injured, the court will say
the intent requirement is met
 A debt resulting from an intentional wrongful act that causes injury is not dischargeable in
bankruptcy, even if the person injured was not the same person whom the debtor intended to
harm.

SAME VICTIM, DIFFERENT INTENTIONAL DIFFERENT VICTIM, SAME INTENTIONAL


TORT TORT
E.g. intended assault of P, accidental battery of E.g. intended battery of P, accidental battery of
P A
CASE: NELSON V. CARROLL CASE: IN RE WHITE
 Attempting to collect a debt, Nelson  White attempted to shoot Tipton but

49
motioned to hit Carroll with the gun, accidentally shot Davis
but accidentally shot him  Court: victims of certain acts that were
 Court: transferred intent permits intended to injure others may sue even
mixing and matching among other though they were not among the
intentional torts intended victims
DIFFERENT VICTIM, DIFFERENT FROM THINGS TO PERSONS
INTENTIONAL TORT E.g. intent to harm chattel, accidentally harm P
E.g. intended assault of P, accidental battery of CASE: LYNN V. BURNETTE
A  Burnette meant to hit the tires of
CASE: IN RE WHITE Lynn’s car, accidentally shot Lynn
 Attempting to scare (assault) Tipton,  Court: this case sounded more like a
White accidentally shot (battery) Davis negligence claim than a battery claim
 Court: tortfeasor is held liable to actual O Perhaps the fact that the case
victim for actions intending harm to a was brought on a charge of
potential victim negligence because the Statute
of Limitations on battery had
run influenced the Court’s
decision.

(iv) Standard Defenses to Battery and Assault


 Some defenses can be raised against a claim of battery or assault that assert that the alleged tortfeasor was
privileged to act as they did, even though their actions were prima facie tortious.
o These privileges are justifications, not excuses.
 The burden of pleading and proving an affirmative defense rests with the alleged tortfeasor. The longstanding rule
is that comparative fault does not apply in cases of battery and assault, hence the defenses of CONSENT, SELF-
DEFENSE, and DEFENSE.

(a) Consent
 A P cannot prevail b/c agreed to endure an act that would be tortious, such as bodily contact, apprehension of
contact, or confinement
o Alleged tortfeasor must prove consent
o P can try to prove a lack of consent
 even when a court determines that the victim has expressly or implicitly consented to some harmful or
inappropriate contact, the question remains as to whether the contact in question was of the sort to which the
plaintiff consented.
o Keep an eye on the scope of consent
 Koffman v. Garnett (Va.2003)
o Facts: a 13 year old who was playing football for the first time. The coach was upset with the team’s
performance and used Koffman to demonstrate proper tackling techniques. Considerable size
difference – Garnett was 120 pounds heavier than the 144 pound Koffman, and was in a position of
authority. Claim brought against Garnett on assault and battery
o D’s Arg: Koffman implied consent to the type of injury by agreeing to play football
o Held: Court determined that Garnett’s actions exceeded the scope of consent. Given the disparity in
size, Garnett’s actions were imprudent and irresponsible, and was therefore negligent
 The claim of assault was dismissed because apprehension of injury happened at the same
time as the alleged battery

50
 The question of battery (i.e. whether Koffman consented) is a jury question, and it is
remanded
o Assault and battery in Virginia
 Assault consists of an act intended to cause either harmful or offensive contract with another
person or apprehension of such contact, and that creates in that other person’s mind a
reasonable apprehension of an imminent battery
 Our court says this cannot proceed
o No apprehension during the situation
 Battery= raising up and slamming down
 The tort of battery is an unwanted touching which is neither consented to, excused, nor
justified
 This tackling from a coach he did not consent to should go to a jury
o The dissent’s position is radical in that it says we should dismiss battery claim before it even goes to
jury
 Most states have consent as affirmative defense
o Virginia puts consent as part of prima facie case

(b) Self-Defense and Defense of Others


 The law gives you a privilege to use certain force when you are defending yourself or others from imminent
physical harm
 Typical self-defense requirements
o Reasonable belief that force is necessary to avoid imminent injury (e.g. physical harm; inappropriate
touching) to oneself or another.
o Force must be proportional to the perceived threat.
 Haeussler v. DeLoretto (Cal.App.1952)
o Facts: This was the case about the dog who went over to the neighbor’s property, and the owner went
over to get it. The owner berated the neighbor about repeatedly keeping the dog. De Loretto urged
Haeussler to back away and leave, but Haeussler was waving his arms around and stepped towards
De Loretto. De Loretto knew that Haeussler had a violent history and had attacked De Loretto’s
friends in the past. De Loretto hit and pushed Haeussler away before closing the door. Haeussler
brings claim on battery
o Held: The court utilized the facts to determine that yes, there was a reasonable assumption of bodily
harm, and yes, the response was proportionate
 force in defense of himself
o In order to defeat self-defense claim, the P must prove by a preponderance of evidence that D used or
attempted to use willfull and unlawful force upon the person or P
 Restatement 2nd § 65: Self-Defense by Force Threatening Death or Serious Bodily Harm
o (1) Subject to the statement in Subsection (3), an actor is privileged to defend himself against another
by force intended or likely to cause death or serious bodily harm, when he reasonably believes that
 (a) the other is about to inflict upon him an intentional contact or other bodily harm, and that
 (b) he is thereby put in peril of death or serious bodily harm or ravishment, which can safely
be prevented only by the immediate use of such force.
o (3) The privilege stated in Subsection (1) does not exist if the actor correctly or reasonably believes
that he can with complete safety avoid the necessity of so defending himself by

51
 (a) retreating if attacked in any place other than his dwelling place, or in a place which is also
the dwelling of the other . . .
 Florida Statute 776.013
o (1) A person who is in a dwelling or residence in which the person has a right to be has no duty to
retreat and has the right to stand his or her ground and use or threaten to use:
 (b) Deadly force if he or she reasonably believes that using or threatening to use such force is
necessary to prevent imminent death or great bodily harm to himself or herself or another or
to prevent the imminent commission of a forcible felony.
o (2) A person is presumed to have held a reasonable fear of imminent peril of death or great bodily
harm to himself or herself or another when using or threatening to use defensive force that is intended
or likely to cause death or great bodily harm to another if:
 (a) The person against whom the defensive force was used or threatened was in the process of
unlawfully and forcefully entering, or had unlawfully and forcibly entered, a dwelling,
residence, or occupied vehicle…; and
 (b) The person who uses or threatens to use defensive force knew or had reason to believe
that an unlawful and forcible entry or unlawful and forcible act was occurring or had
occurred.

(c) Defense and Recapture of Property


 The general rule is that a possessor of land may not set up traps that pose a serious threat of life and limb to a
trespasser. The possessor may set up reasonable means to repel a trespasser, but nothing outweighs the right
to life and limb.
 Katko v. Briney (Iowa1971)
o Facts: This was the case about the people whose unoccupied farmhouse kept getting burglarized, so
they set up a trap with a shotgun. There were no warnings of the presence of the gun, and it was
intentionally concealed. There was some debate as to whether the gun should aim at the stomach or
the leg of an intruder
o Held: The court upheld the general rule
 Cannot set up a spring gun trap to protect personal property against trespassers and thieves
 Personal rights are given a higher value under the law than property rights
 Can only use deadly force to protect against trespassers when to prevent a felony of violence
where human life is in danger

B. Infliction of Emotional Distress


(i) Intentional Infliction of Emotional Distress (IIED)
 Intentional Infliction of Emotional Damages became developed as a kind of “gap-filler” tort, when traditional
intentional torts like battery, assault, and false imprisonment were not met, yet justice was still required.
o As such, a claim for IIED can only be brought if the other intentional torts are not met first. It cannot be
used as a piggyback claim to be brought alongside another intentional tort.
o The actions must be truly outrageous- mere insults or indignities are insufficient to establish IIED
 Restatement 2nd section 46: outrageous conduct causing severe emotional distress
o 1) One who by extreme and outrageous conduct intentionally or recklessly causes severe emotional
distress to another is subject to liability for such emotional distress, and if bodily harm to the other results
from it, for such bodily harm.
 Note that the Restatement 3rd has a virtually identical provision (see p. 703).

52
 Note also the book’s working definition of recklessness: “a failure to heed a very obvious and
very significant risk of serious injury.”
 the emergence of IIED
o Wilkinson v. Downton, 2 Q.B. 57 (1897)
 Plaintiff alleged that the defendant falsely informed her that her husband had been “smashed up”
in an accident and suffered two broken legs.
 Evidence that plaintiff suffered nausea and other physical side effects from the experience.
 Defendant liability upheld he caused her physical harm by having “wilfully done an act calculated
to cause [such] . . . harm.” Cause of action unspecified.
o Nickerson v. Hodges, 84 So. 37 (La. 1920)
 After pot of gold April Fool, plaintiff sought compensation for expenses, lost wages, mental and
physical suffering, and humiliation.
 Verdict for defendant reversed; $500 awarded. Cause of action unspecified.
 Dickens v Puryear (NC 1981)
o Facts: This was the case about the man who was having sexual relations with a 16 year old, as well as
sharing alcohol, marijuana and cigarettes with her. Her parents set a trap for Dickens; Dickens was then
tied up and beaten by a group of men over the course of several hours. At least 5 times, they voted
whether to kill or castrate Dickens. Dickens was finally released, and was directed to leave town or else
he would be castrated
o Held:
 No IIED: Beatings – battery, Handcuffing – battery; false imprisonment, Hair-cutting – battery,
Threat of castration/death – assault
 IIED: Conditional Death Threat
o The question of foreseeability does not arise in the tort of IIED
o A threat for the future harm did inflict serious mental distress; therefore, it is actionable, if at all, as an
intentional infliction of mental distress
o A D is liable for IIED when he desires to inflict severe emotional distress, and also where he knows that
such distress is certain, or substantially certain, to result from his conduct. It applies also where he acts
recklessly… in deliberate disregard of a high degree of probability that the emotional distress will follow.
o IIED in North Carolina
 This tort . . . consists of:
 (1) extreme and outrageous conduct,
 (2) which is intended to cause and does cause
o the tort may also exist where defendant's actions indicate a reckless indifference
to the likelihood that they will cause severe emotional distress.
 (3) severe emotional distress to another.
o Recovery may be had for the emotional distress so caused and for any other bodily harm which
proximately results from the distress itself.
 Hunt v State (Del. 2013)
o Facts: This was the case of the state trooper who was hired by the school to educate high school students.
Officer Pritchett had learned that one of the fifth graders had stolen an autistic student’s money – he went
to the student’s mother and asked for permission to lure him into a trap. Pritchett turned to Hunt (who sat
next to the autistic student) to try to get AB to admit to taking the money. Warned Hunt ahead of time that
he was not in any trouble and instructed him just to deny that he took the money. Once inside the room,
Pritchett allegedly repeatedly told Hunt that he had the authority to arrest him, and that he could send him

53
to a place where children are treated like criminals. Basically, reduced him to tears, to cause AB to admit
that he stole the money
o Held:
 Battery – none – the light touch to Hunt was not offensive or socially unacceptable
 False Imprisonment – enough to show that Hunt was detained by Pritchett, given that the door
was closed, and that Pritchett was in a position of authority
 Also, he did not tell Hunt that he was free to leave if he desired
 A reasonable jury could find this to be false imprisonment
 A reasonable jury could find Pritchett’s behavior to be outrageous and reckless
 Pritchett was only supposed to work with high school students
o Takeaways
 Example of fact scenario that the court says a reasonable jury could find was extreme and
outrageous
 The court doesn’t say that allegations are enough, but reasonable jury could find
 Mental state inadequate
 Low bar for this court? Intent?
o No intent to cause IIED
o A claim for intentional infliction of emotional distress (IIED) requires proof that the defendant
intentionally engaged in extreme or outrageous conduct that caused severe emotional distress
 “Outrageous behavior,” as required for intentional infliction of emotional distress, is conduct that
exceeds the bounds of decency and is regarded as intolerable in a civilized community.
 It is for the court to determine, whether the D’s conduct may reasonably be regarded as
so extreme and outrageous as to permit recovery for intentional infliction of emotional
distress, but, if reasonable minds may differ, the question of whether the conduct is
extreme and outrageous is for the jury.
o Requirements for Hunt’s Section 1983 claim
 That he was deprived of a federal right; *
 That Pritchett was acting under the color of state law;
 That Pritchett’s conduct is not protected by qualified immunity, i.e.:
 That Pritchett’s conduct violated a “clearly established” right, i.e.
 That it would have been “clear to a reasonable [official] that his conduct was unlawful in
the situation he confronted.”
 Hunt is alleging a violation of the 4th Amendment (“The right of the people to be secure in their
persons . . . against unreasonable searches and seizures, shall not be violated . . . “).
o Comment & illustration: 2nd Restatement 46
 e. The extreme and outrageous character of the conduct may arise from an abuse by the actor of a
position, or a relation with the other, which gives him actual or apparent authority over the other,
or power to affect his interests
 In particular police officers, school authorities, landlords, and collecting creditors have
been held liable for extreme abuse of their position.
 Illustration:
 A, the principal of a high school, summons B, a schoolgirl, to his office, and abruptly
accuses her of immoral conduct with various men. A bullies B for an hour, and threatens
her with prison and with public disgrace for herself and her parents unless she confesses.
B suffers severe emotional distress, and resulting illness. A is subject to liability to B for
both.
54
o Comment j. to Restatement 2nd 46
 j. The rule stated in this Section applies only where the emotional distress has in fact resulted, and
where it is severe. Emotional distress passes under various names, such as mental suffering,
mental anguish, mental or nervous shock, or the like. It includes all highly unpleasant mental
reactions, such as fright, horror, grief, shame, humiliation, embarrassment, anger, chagrin,
disappointment, worry, and nausea. . . . Complete emotional tranquility is seldom attainable in
this world, and some degree of transient and trivial emotional distress is a part of the price of
living among people. The law intervenes only where the distress inflicted is so severe that no
reasonable man could be expected to endure it. . .

(ii) Negligent Infliction of Emotional Distress


 NEGLIGENT INFLICTION OF EMOTIONAL DISTRESS is not physical damages and distress, or when defendant causes
plaintiff to cause a discrete physical harm (like a heart attack). NIED refers to the emotional well-being of a
plaintiff, causing trauma, but not a discrete physical injury.
 RULE: there is no duty to take care to avoid causing emotional distress.
o There are some CAVEATS:
 Where the defendant’s conduct goes beyond carelessness to intentionality or recklessness; and
 If emotional harm suffered by plaintiff is parasitic on physical injury suffered due to the activities
of defendant, it is actionable.
 THE EXCEPTIONS TO THE GENERAL RULE:
o If a special relationship exists between parties that obligates one to avoid inflicting emotional harm; and
o Where the defendant acts carelessly so as to place plaintiff in imminent bodily harm of which plaintiff is
contemporaneously aware but bodily harm does not materialize.
 Must be an objective standard of imminent danger
 Must be caused by negligence, not intent
 Caveat – a plurality rule, not a majority rule, is the growing acceptance
 Negligence claims w/ physical injury satisfying prima facie injury element
o Plaintiff also includes pain & suffering as part of injury alleged
o These are said to be parasitic on the physical injury
o Example: Walter.
o Duty invoked in these cases is a duty to take care not to cause physical injury
o These are called regular negligence claims, as opposed to NIED

(a) Physical impact rule


o Allows recovery for mental distress if P can show that he had suffered a contemporaneous physical
impact
 the idea that a physical contact is needed for emotional distress claims to be parasitic and
recovered upon
o Wyman v. Leavitt (Me. 1880)
 Facts: P sued for emotional injury caused by fear of her own life from the blasting of rocks on a
neighboring plot of land
 Held: ruled against P; stated no physical harm was done to her or her family, and upholds the
general rule
 Wyman can’t recover

55
o Her emotional distress claim isn’t parasitic to the property damage- her husband
owns the property (not her), and reflects traditional reluctance to allow recovery
for ED claims parasitic to claim for negligent property damage
 If damage to property had been intentional, or if she owned the property,
she may have been able to recover
 What Wyman finds in the law
o Personal injury claims for “mental pain” following “bodily pain.”
o In battery cases, claims not just for pain but for the “mental condition”
arising from the insult of the defendant’s blows.
o Assault claims, which can be made even absent physical injury.
o In property torts, claims relating to “injury to the feelings of the plaintiff,” when
the defendant is motivated by “wantonness or malice, or a willful disregard of
others’ rights.”
o But no negligence cases where “mental suffering alone,” absent physical injury,
is alleged.
o One case (Johnson) saying damages in a negligence suit can’t include “pain of
mind aside and distinct from bodily suffering.”
 Johnson v. Wells Fargo (cited in Wyman)
o “How can [damages for mental suffering] be estimated in money? The mental
agony of a timid woman would be entirely different from that of a bold man. No
two cases could be weighed in like scales. To properly estimate such a cause of
damage, the door must be opened to the realms of philosophy, physiology, and
psychology . . . “
 Fears were not relevant under the law

(b) Zone of danger test


o The zone of danger rule imposes a duty to take care against causing distress to another if (all three must
be met):
 P must be w/in the zone of physical danger from D’s carelessness (objectively determined)
 P must have a physical manifestation of her distress
 P’s fear of being physically injured must be the cause of her fright/ emotional distress
o Robb v. Pennsylvania R.R. Co. (Del. 1965)
 Facts: This is the case where the woman’s car stalled on train tracks. She narrowly escaped being
smashed by an incoming train . Robb suffered considerable physical damages as a result of her
fright – it affected her nervous system, causing a cessation of lactation; she was unable to
maintain her horse-breeding business
 Held: The Court rejected the old IMPACT RULE and adopts the zone of danger test
 Decides that the Impact Rule is too arbitrary
 Where negligence proximately caused fright, in one within the immediate area of
physical danger from that negligence, which in turn produced physical consequences
such as would be elements of damage if a bodily injury had been suffered, the injured
party is entitled to recover under an application of the prevailing principles of law as to
negligence and proximate causation.
o P is allowed to recover for her mental distress
 Impact rule thrown out

56
o Robb on justifications for ‘impact rule’
 Since fright alone does not give rise to a cause of action, the
consequences of fright will not give rise to a cause of action.
 The physical consequences of fright are too remote, and the requisite
causal connection is unprovable (i.e. no proximate cause).
 Public policy and expediency demand that there be no recovery for the
physical consequences of fright in the absence of a contemporaneous
physical injury.
 Adopts the Zone of danger test
o Consolidated Rail Corp. v. Gottshall (U.S. 1994)
 Facts: This was the case about the worker who saw his friend suffer a fatal heart attack because
the project was behind schedule; the workers were overworked in the heat without any breaks. He
tried for forty minutes to resuscitate his friend. The body was in plain sight as the workers were
forced to return to work. The trauma resulted in Gottshall suffering from severe depression,
causing him to lose significant amounts of weight, unable to sleep, etc. Another claim by Carlisle,
who claimed that the high stress of his work caused him to suffer injuries
 Held: the Zone of Danger test applied, that there can be NIED without physical manifestation
requirement for Gottshall
 Carlisle’s case was dismissed – they said he was merely claiming to be overworked rather
than having suffered from NIED
 Court adopts the zone of danger test
o The primary focus of the statute is allowing RR employees to recover for
physical injuries- so mental injuries should be given second class status
o Policy reasons for limiting liability via the zone of danger test
 Concern about trivial claims
 Convern about fraudulent claims
 Concern about unlimited and unpredictable liability
 NIED tests reviewed by Gottshall
o “Physical impact” test.
 Plaintiff must allege a contemporaneous physical impact (no matter how
slight) or injury due to defendant’s conduct.
o “Zone of danger” test.
 Plaintiffs must have sustained a physical impact as a result of a
defendant’s negligent conduct, or have been placed in immediate risk of
physical harm by that conduct.
 SCOTUS thinks this should be applied
 FELA & NIED
o “Every common carrier by railroad while engaging in commerce . . . shall be
liable in damages to any person suffering injury while he is employed by such
carrier in such commerce, for such injury . . . resulting in whole or in part from
the negligence of any of the officers, agents, or employees of such carrier. . .”
o Elements of NIED claim under FELA:
 Duty: to avoid inflicting emotional harm to those in the zone of danger;
 Breach: deviating from ordinary person’s standard of care;
 Causation: actual & proximate;
 Injury: emotional harm (perhaps with physical symptoms).
57
Liability without Fault and Products Liability
A. Property Torts and Ultrahazardous Activities
(i) Property Torts
(a) Trespass to land: prima facie case
 Trespass to land: trespass to land is the intentional unauthorized entry onto the land of another
o Trespass is an intentional tort. The intent is the intent to enter the land.
 Restatement Second § 158
o One is subject to liability to another for trespass . . . if he intentionally
 (a) enters land in the possession of the other, or causes a thing or a third person to do so, or
 (b) remains on the land, or
 (c) fails to remove from the land a thing which he is under a duty to remove.
o Comment e. Conduct which would otherwise constitute a trespass is not a trespass if it is privileged.
Such a privilege may be derived from the consent of the possessor, or may be given by law because
of the purpose for which the actor acts or refrains from acting.
 Some details of what constitutes trespass
o Trespass can involve invasion of the property by the defendant herself or by tangible matter set in
motion by the defendant (e.g., animals, waste, water, rocks, construction, etc.)
o Trespass can include failure to leave (or remove an object) after permission expires or is withdrawn,
see Vincent, or failure to remain within the scope of the permission to enter. See Leffler.
o The invasion can be minimal. See Hulle v. Orynge (1466) for trespass constituting thorns falling onto
land.
o Plaintiffs are not required to show damage or loss of value.
o Trespass can potentially consist of invasions below, on, or above land.
 Burns Philp Food Inc. v. Cavalea Cont’l Freight, Inc. (7thCir.1998)
o EVEN IF YOU DON’T KNOW YOU’RE COMMITTING TRESPASS, YOU CAN STILL BE
HELD LIABLE
 Intro to possible defense of consent
o Facts: This was the case where Nabisco sold some land around Chicago, and there was an issue with
the recording of the sale, so Burns and Cavalea thought that BPF owned land that actually belonged
to Cavalea. BPF installed a fence that crossed into Cavalea’s land. When BPF found out about the
error in the recording documents, it notified Cavalea, asking for money for taxes paid for Cavalea’s
land. Cavalea refused and removed BPF’s fence, making a counterclaim against BPF for trespass
o Held: that there was a trespass – despite neither side knowing of the true ownership and BPF
innocently building the fence, it was still an intrusion on the land. The damages are the losses from
the use of the land cut off by BPF
 Trespass is entry without consent; while the consent lasts there can be no trespass, and
therefore no legal remedy
 Trespass is a strict liability tort and an obligation to notify the intruder is inconsistent with the
idea of strict liability

(b) Trespass and necessity


 General rule:
o under the defense of necessity, D has a privilege to harm the property interest of P where this is
necessary in order to prevent great harm to 3rd persons or to the D herself
58
o necessity is an incomplete privilege
 Public necessity:
o if interference with the land or chattels of another is necessary to prevent a disaster to the community
or to many people, the privilege is that of public necessity.
o Here, no compensation must be paid by the person doing the damage
 Private necessity:
o If a person a person prevents injury to himself or his property, or to the person of property of a third
person, this is protected by the privilege of private necessity, if there is no less damaging way of
preventing the harm
 Actual damage
 Where the privilege of private necessity exists, it will be a complete defense to a tort
claim where P has suffered no actual substantial harm.
 But if damage occurs, P must pay for the damage she has caused
 Owner may not resist
 The main purpose of the doctrine of private necessity is to prevent the person whose
property might be injured from defeating the exercise of the privilege
o Vincent v. Lake Erie Transp. Co. (Minn.1910)
 NECESSITY CAN LIFT LIABILITY OF TRESPASS, IF YOU CAN SAY THE INVASION
WAS DONE TO HELP HEALTH OR LIFE
 Facts: This was the case about the ship that was tied at a dock. It was there to unload and was
supposed to leave; it tried, but was unable. If the ship did untie from the dock, it may have
been tossed about in the storm, maybe thrown onto a soft mud bank. Ship, tied to the dock,
was bouncing back and forth and caused damage to the dock, despite the best efforts of the
crew to minimize the damage
 Held: Court determined that there was a trespass – it was a choice by defendant to remain tied
at the dock
 The D prudently availed himself of P’s property for the purpose of preserving its own more
valuable property- even so P’s are entitled to compensation for the injury done
 D trespassed onto P’s dock, but it was justified by necessity
o Necessity doesn’t provide an absolute privilege
 The privilege is incomplete b/c the ship must pay for damages it
inflicts by being there
 Unjust enrichment
o If you benefit at someone else’s cost, you have to
pay back

(c) Conversion and trespass to chattel (T/C)


1. Trespass to chattel (T/C) (NOT ON EXAM)
o Look for T/C whenever one person intentionally interferes with another’s possession of a thing
o T/C is the intentional interference with another’s possessory interest in a chattel, resulting in damage
to that interest
o The intent to take possession or otherwise affect the chattel. D’s belief about his right to do the act, or
about who holds title, is not part of the requisite intent.
 Mistake is never a defense to T/C.
 The following types of mistake (even if reasonable) will not be a defense

59
o D believes the chattel already belongs to D
o D believes that X has title to the object, and buys the object from X when it
really belongs to P
o D is a creditor who wrongly thinks that he has the right to repossess P’s
chattel
 Distinction between intent and accident
 If D doesn’t intend to even make contact with P’s possession, and this contact
happens by accident, there is no T/C
o The degree of interference
 Damage is deemed to take place even if the only damage is a temporary one to P’s right of
possession and if the item is returned unharmed

2. Conversion
 Restatement 2nd § 222A: What Constitutes Conversion
o (1)  Conversion is an intentional exercise of dominion or control over a chattel which so
seriously interferes with the right of another to control it that the actor may justly be required
to pay the other the full value of the chattel.
o (2)  One who effectively prevents the exercise of intangible rights of the kind customarily
merged in a document is subject to a liability similar to that for conversion, even though the
document is not itself converted.
 How to approach a Conversion Q
o Look for at the same time looking for T/C. Conversion is more “serious than T/C”
o occurs when the D so substantially interferes with the P’s possession or ownership of goods
that it is fair to require the D to pay the property’s full value
o Conversion is an intentional tort
 The intent to take possession or otherwise affect the chattel.
 D’s mistake about the right to possess, or about who has title, doesn’t negate
his intent
o Different ways to commit conversion
 Acquiring possession: D takes possession of the property from P
 Transfer to 3rd person: D transfers a chattel to one who is not entitled to it
 Withholding good: D refuses to return goods to their owner, if the regusal lasts for a
substantial time
 Destruction: D destroys the goods, or fundamentally alters them
o Intangible property such as e-files
 If the property in Q is intangible, point out that the item must be property of the sort
required for these torts under state law
 In most courts e-files are deemed to be “property” that can be trespassed or
converted, so if D permanently deprives P of access to files P owns, D will be liable
 Thyroff v. Nationwide Mut. Ins. Co. (N.Y.2007)
 Facts: This was the case about the guy who had an agreement with
Nationwide – he could use their computers and their systems. After
Nationwide terminated the agreement, they took back the computers
and cut off Thyroff’s access to the servers. Thyroff had saved all of
his information on the servers and needed it

60
 Held: The Court ultimately held that conversion should cover
intangibles – prior to this, they only covered tangible property
 Merger Doctrine (majority opinion): intangibles may support
an action if there is a token physical representation, such as a
stock certificate
 There is no fault requirement
 claim for the conversion of electronic data is cognizable
under NY Law
o If P is successful with her tort suit, a forced sale occurs - D buys the item for the value it had
at the time of the conversion
 If it’s conversion, the value of the item is the sole measure of damages
 What conversion covers
o Historically, only tangible personal property (so not land, for example)
o The merger rule allowed some expansion. Under it, courts have held that:
 Youcanconvertintangiblepropertyinterestsbyexercisingdominionoveradocumentor
other physical item that represented that interest (see share certificate; master
recording)
 You can convert intangible property interests (e.g. shares)if they do, or would
normally, have a physical token (e.g. share certificate) (See Restatement 2nd), or are
the kind of thing that could have such a physical token.
o For a broad reading of merger, see Kremen, which says that in California merger means at
most some connection to a document or tangible object. (Property was a domain name;
“document” was an electronic database of domain names).
o Some courts have expanded conversion to electronica without relying on merger.
o Note that Thyroff limits its holding to “electronic records that were stored on a
computer and were indistinguishable from printed documents.”
 Ancestors of conversion
o Robbery & larceny – both private actions. No damages permitted, only return of the
property.
o Trespass de bonis asportatis – limited damages if the defendant offered to return the
property to its rightful owner.
o In late 15th Century, trover arrived, to fill the gap left by the trespass cause of action. It was
aimed at a person who had found goods and refused to return them to their owner. Damages
in the form of the full value of the chattel at the time and place of the conversion.
o Trover came to cover additional misappropriations, including thefts, and slowly gave way to
the broader tort of conversion.

3. T/C versus Conversion


o Distinguishing Conversion and T/C
 Conversion only occurs when the injury to P’s interests is so severe that it is appropriate to
make D pay for the whole value of the item as opposed to damages for just the interference
 Factors the court considers when considering conversion
 Extent and duration of dominion: the extent and duration of the dominion (the
greater/longer, the more likely it’s conversion)

61
 Good or bath faith: if D thinks seller has good title- this suggests T/C rather than
conversion b/c D has behaved in good faith
 Degree of harm: degree of harm greater--> conversion
 Inconvenience and expense caused to P
o Why the distinction is important
 T/C, D just pays for the damages
 Conversion, there’s a forced sale

(d) Consent (with notes on media trespass and on defenses other than consent)
 Types of consent
o Express consent: if P expressly consents to an intentional interference with his person or property, D
will not be liable for that interreference
o Implied consent: existence of consent may also be implied from P’s conduct, from custom, or from
the circumstances
 Objective manifestation: it is the objective manifestations by P that count- if it reasonably
seemed to one in D’s position that P consented, consent exists regardless of P’s subjective
state of mind
 Lack of capacity: consent will be invalidated if P is incapable of giving that consent, b/c she is a child,
intoxicated, unconscious, etc.
o Consent as a matter of law
 But even if P is incapable of truly giving consent, consent will be implied as a matter of law
if these factors exist
 P is unable to give consent
 Immediate action is necessary to save P’s life or health
 There is no indication that P would not consent if able, and
 A reasonable person would consent in the circumstances
 Exceeding scope
o Even if P does consent to an invasion of her interests, D will not be privileged if he goes substantially
beyond the scope of that consent
 Limits to consent
 Geographic
 Temporal (Vincent)
 Purpose (Copeland)
o Copeland v. Hubbard Broadcasting, Inc. (Minn. Ct. App. 1995)
 Facts: This was the case about the veterinary student who was also working for the network –
as part of an investigation, she followed the vet. The owners of the home allowed the vet and
the student on a home checkup on their cats – the student secretly recorded them.
 D’s Arg: Defendant claims consent (express and implied) was given to the student to be on
the premises
 The claim of trespass states that she was permitted on as a student, not as a journalist
 Held: scope of consent can be exceeded in manners not limited to the geographic scope
 P’s can bring claim of trespass- student violated the scope of P’s consent
o P only gave consent to allow a veterinary student to accompany the
veterinarian

62
(e) Nuisance
 2 types of nuisance: public and private
o We are only concerned with private nuisance
 Nuisance: a working definition
o A defendant commits a (private) nuisance against a plaintiff if he engages in conduct that causes a
continuing and unreasonable interference with the plaintiff’s use and enjoyment of property in which
she has a possessory interest.
 Nuisance v. Trespass
o nuisance requires the defendant’s conduct to have caused unreasonable interference with another’s
use and enjoyment of land, whereas trespass exists even for trivial physical invasions.
 Liability in trespass is also more one-sided in that there is little, if any, consideration given to
the value of trespassory activity in determining whether a trespass has occurred.
 In nuisance, by contrast, defendant’s interest in pursuing the offending activity often
enters into the analysis of whether a nuisance exists.
 Key Points for Nuisance
o No intent to harm is needed
o Need voluntary act on part of D
o Do not need to show D was unreasonable in his or her behavior
 Rather, need to show that what happened to your enjoyment of the land was unreasonable
 Sturges v. Bridgman (C.A. 1879)
o Nuisance is context specific  have to see what the dominant industry is
o To determine if a thing is a nuisance, consider the circumstances of the nuisance
o Facts: This is the English case about the doctor extended a part of his home that was formerly a
garden to become a consulting room. The now-consulting room abutted against the neighbor’s land,
which was a confectionary. Doctor claimed that the noise was interfering with his use of the indoor
space including the quiet needed for consultations. He sought an injunction
o Held: P is allowed to recover. he is not estopped from making a complaint just because he didn’t
complain until he built the addition
 Court says the fact that D has until now been at liberty to use the mortars can’t be regarded as
evidence that P acquiesced in their use as to forfeit his right to complain about them now
 “Coming to the nuisance” – an alternative view that will typically fail
 Court points out that the circumstances, this is what was expected in the neighborhood, which
had a lot of small scale industry
o The Coase Theorem:
 Should the court confer a right to quiet on defendant as a right to make noise?
 Some think that the courts should attempt to determine directly whether the plaintiff’s use or
defendant’s use has the higher value, but courts are often not able to make these judgments,
and assuming no impediments, the market, not the court, would be able to decide
 Restatement (2nd) § 840D: Coming to the Nuisance
o The fact that the plaintiff has acquired or improved his land after a nuisance interfering with it has
come into existence is not in itself sufficient to bar his action, but it is a factor to be considered in
determining whether the nuisance is actionable.
 Penland v. Redwood Sanitary Sewer Serv. Dist. (Or. Ct. App. 1998)
o Whether a condition constitutes a nuisance depends on its effect on “an ordinarily reasonable person,
a normal person of ordinary habits and sensibilities”

63
 Once a nuisance has been established, the court may refuse an injunction in certain cases
where the hardship caused to the defendant by the injunction would greatly outweigh the
benefit resulting to the plaintiff.
 Even if courts find nuisance, it remains in the court’s discretion whether to order an
injunction
 Compare the benefit to plaintiffs with the hardship to the defendant
o Facts: This was the case about the sewage recycling plant that changed its methods as to be more
environmentally friendly. The smell was so bad that entire neighborhoods were unable to hang out
outside
o D’s Arg: they followed all of the govt. regulations
o Held: this was a nuisance and an injunction is granted
 Five factors to assess nuisance (not clear if all 5 must be met)
 Location of the claimed nuisance
 Character of the neighborhood
 Nature of the thing complained of
 Frequency of the intrusion
 Effect upon the plaintiff’s enjoyment of life, healthy and property
 Granting of an injunction is up to the discretion of the court Our court says that an
injunction is proper
 Overcome if hardship to the defendant greatly outweighs the benefit to the defendant
 There were no great and undue hardships here:
o Defendant chose to expand its operations, knowing of the complaints
o Costs can manageably be spread across ratepayers - $5 per person
o Oregon standard for granting an injunction
 The court may refuse an injunction where the hardship caused to the D by the injunction
would greatly outweigh the benefit resulting to the P
 The injunction does not issue as a matter of absolute or unqualified right but is subject to the
sound discretion of the court
 Boomer v. Atlantic Cement Co. (N.Y. 1970)
o In NY, such a nuisance will be enjoined although marked disparity be shown in economic
consequence between the effect of the injunction and the effect of the nuisance.
o Facts: This was the case about the cement factory that created smoke, air pollution, vibrations, etc.
The cement making operations apparently damaged nearby properties of the plaintiffs. Total damage
to the property of the plaintiffs found to be relatively small in comparison to the value of the
defendant’s operations
o Held: Injunction should be granted.
 NY Law states that nuisance victims are entitled to an injunction even if there is a marked
disparity in the costs and benefits of the injunction
 But in the circumstances, the injunction could have significant damages to the
defendant, so the court settles with the payment of “permanent damages” as a means
of spurring the development of more environmentally friendly technologies
 Issues of public policy.
 A court should not try to regulate air control and pollution on its own as a by-product
of private litigation and it seems manifest that the judicial establishment is neither

64
equipped or prepared to lay down and implement an effective policy for the
elimination.
 Boomer’s version of pre-Boomer law
o Where a nuisance has been found and where there has been any substantial damage shown by the
party complaining an injunction will be granted [even if marked disparity is shown in economic
consequence between the effect of the injunction and the effect of the nuisance....]

(ii) Ultrahazardous (Abnormally Dangerous) Activities


 For ULTRAHAZARDOUS or ABNORMALLY DANGEROUS ACTIVITIES, there is strict liability.
o It is very limited in scope, and these are Abnormally Dangerous Activities:
o Only a handful of categories in which this form of strict liability applies
o The most well-established:
 (1) Wild animals;
 (2) Engagement in activities involving routine use of explosives or the storage of vast amounts of
potential energy.
 It is important to remember that the strict liability of ultrahazardous activities applies only to bystanders, not
participants.
o This eliminates the ability to expressly or implicitly assume the risk. In terms of limitations, defenses
such as comparative fault and assumption of the risk will typically apply on the same terms as they do for
negligence claims.
 The RESTATEMENT has a two-part test to determine if something is Abnormally Dangerous:
o Step 1: It creates a foreseeable and highly significant risk of physical harm even when reasonable care is
exercised by all actors; and
o Step 2: It is not an action of common usage
 Rylands v. Fletcher, All E.R. 1 (1868)
o Created strict liability only in the sense of relieving the P of the usual burden of proving fault, thereby
subjecting the D to liability unless it could prove lack of fault
o Facts: This was the classic case where Ryland had a mine and built a reservoir with Fletcher’s knowledge
and assent. The reservoir leaked into underground shafts and flooded Fletcher’s coal mine
o Held: the plaintiff could recover without proving that defendant intended contact, committed a nuisance,
or was at fault
 A defendant would only be able to excuse himself if it was an “act of God” – something so wildly
unforeseeable and uncontrollable
 One who keeps something on his land that could cause damage is responsible for damages caused
by its escape to an innocent person
 Strict liability imposed on D b/c his storage of water on his property created a very significant
risk of property damage to his neighbor, which was in the end realized
 D’s actions constituted an ultrahazardous activity
 Klein v. Pyrodyne Corp. (Wash. 1991)
o Must still worry about causation, even when dealing with strict liability
o Can have strict liability and negligence claim at first time
o Facts: This was the case about the fireworks display at a state fair. A firework shell hit a plaintiff. The
plaintiff claimed negligence – that an employee wasn’t paying attention, causing a mortar to fall, setting
off the firework and injuring the plaintiff/ Defendant claimed fault on the part of the manufacturer,
resulting in a defective piece that went off early

65
 RCW 70.77.285
 [T]he applicant for a permit . . . for a public display of fireworks shall include with the
application evidence of a bond issued by an authorized surety company. The bond shall
be in the amount required by RCW 70.77.295 and shall be conditioned upon the
applicant’s payment of all damages to persons or property resulting from or caused by
such public display of fireworks, or any negligence on the part of the applicant or its
agents, servants, employees, or subcontractors in the presentation of the display. Instead
of a bond, the applicant may include a certificate of insurance evidencing the carrying of
appropriate public liability insurance in the amount required by RCW 70.77.295 for the
benefit of the person named therein as assured, as evidence of ability to respond in
damages.
o Held: this was a dangerous activity, determining that a “common usage” does not impose an
asymmetrical risk; Pyrodyne is strictly liable
 The court strays from the statute and says strict liability should be imposed
 This asymmetrical risk must be reciprocated for it not to be an abnormally dangerous activity –
for example, a car imposes a symmetrical risk to other motorists
 Pyrotechnicians are strictly liable for damages caused by fireworks displays
 The court rejects
 Restatement (2nd) § 522
o One carrying on an abnormally dangerous activity is subject to strict liability for
the resulting harm although it is caused by the unexpectable
 (a) innocent, negligent or reckless conduct of a third person . . .
 Restatement (2nd) § 519
 (1) One who carries on an abnormally dangerous activity is subject to liability for harm to
the person, land or chattels of another resulting from the activity, although he has
exercised the utmost care to prevent the harm.
 (2) This strict liability is limited to the kind of harm, the possibility of which makes the
activity abnormally dangerous.
 The court provides us with a list of ELEMENTS to determine if something is abnormally
dangerous:
 WA (and a majority of other states) have adopted Restatement (2nd) § 520: Abnormally
Dangerous Activities
o In determining whether an activity is abnormally dangerous, the following
factors are to be considered:
 (a) existence of a high degree of risk of some harm to the person, land or
chattels of others;
 (b) likelihood that the harm that results from it will be great;
 (c) inability to eliminate the risk by the exercise of reasonable care;
 (d) extent to which the activity is not a matter of common usage;
 (e) inappropriateness of the activity to the place where it is carried on;
and
 (f) extent to which its value to the community is outweighed by its
dangerous attributes.
 One not enough but you don’t need all of them
o Majority and concurrence disagree about some of the factors
 Majority thinks a, b, c, and d=plus factors
66
 Majority just wants enough plus factors
 Concurrence thinks a and b matters, and f is a minus

B. Products Liability
(i) Introduction
 Products liability: the liability of a seller of a tangible item which, because of a defect, causes injury to its
purchaser, user, or sometimes bystanders
o The liability can be based upon negligence, warranty, or strict tort liability
 These are not new claims, either. MACPHERSON V. BUICK was an early case of products liability in tort law.
o In that case, it rejected the PRIVITY limit on negligence liability for consumer injuries caused by
carelessly made products.
 If a product is carelessly made and is reasonably certain to put life and limb in peril, and if the manufacturer
knows the product will be used by non-purchasers and not be inspected for safety after the sale, then the
manufacturer is under a duty to those to make the product carefully.

(a) Precursors
 STRICT PRODUCTS LIABILITY was encouraged by Justice Traynor in ESCOLA V. COCA-COLA.
o In that case, he advocates for the idea that a manufacturer’s proof of fault is not needed to impose
liability. As with all tort activity, causation (actual or proximate) must be satisfied. Products liability
that results in pure economic loss typically does not fall under strict products liability.
o Traynor seemed to see products liability extending only to a defect in the product that causes harm in
its ordinary use.
 Things that make a product “defective” are:
1. Deviations from the manufacturer’s plan; and
2. Features of the product not meant to be present.
 Escola v. Coca Cola Bottling Co. of Fresno (Cal. 1944)
o Facts: This was the case about the waitress in the restaurant who was restocking the fridge with Coca-
Cola bottles. One of the bottles exploded in her hand, causing significant injuries. When she dropped
the bottle, it did not shatter. Co-workers heard the explosion from 20 feet away. Testimony from
drivers and deliverymen showed that this is not an uncommon occurrence – the frequently found
exploded bottles. The manufacturer of the bottle attests that they test one in every 600 bottles. Court
notes that the old bottles that Coca-Cola takes in to re-use are NOT tested for structural integrity –
possibly one of these injured plaintiff
o D’s Arg: Coca-Cola claimed that they did not have exclusive control of the product at the time of the
accident – the bottles had been sitting on the fridge for a few weeks
 Also, Coca-Cola claims that this kind of injury was not foreseeable or controllable by the
defendant
 Defendant was able to refute the claim for negligence because the plaintiff could not specify
careless acts by the defendant, but not res ipsa loquitur
o Held: The court says res ipsa applies – the plaintiff was passive
 But the defendant did not have control over the bottle
 Res ipsa loquitur does not apply unless (1) defendant had exclusive control of the thing
causing the injury and (2) the accident is of such a nature that it ordinarily would not occur in
the absence of negligence by the defendant.
o TRAYNOR’S CONCURRENCE – very important (the beginnings of products liability)
 A manufacturer should be liable if it:
67
 Places products on the market;
 The product contains a defect when it is placed on the market;
 The manufacturer knows that the product is to be used without inspection; and
 The product causes personal injury during normal use
 Traynor’s concurrence states that the proof of the manufacturer’s fault is NOT required,
hence the “strict” products liability
 Central to Traynor’s concurrence was a concern for DETERRENCE and COMPENSATION
 Strict liability better promotes safe products than negligence does
 In a world of mass production, loss-spreading is achievable and strict liability is
better for spreading losses than negligence

(ii) Basics of a Products Liability Claim


 THE ELEMENTS (PRIMA FACIE CASE):
o P has suffered an injury;
o A sold a product;
o A is a commercial seller of such products;
o At the time that it was sold by A, the product was in a defective condition; and
o The defect functioned as an actual and proximate cause of P’s injury.

NEGLIGENCE V. STRICT PRODUCTS LIABILITY


THE PRIMA FACIE CASE
NEGLIGENCE STRICT PRODUCTS LIABILITY
Injury Injury
Duty Product Sold by Defendant
Breach Defendant is a Commercial Seller of the
Product
Product was Defective at the time
Causation (actual or proximate) Causation (actual or proximate)

(a) What counts as an injury?


 An INJURY can occur to property; injury is not restricted to personal injuries.
o For example, a defective water heater can cause a plaintiff’s house to burn down, and the heater’s
manufacture is subject to liability.
 Economic loss rule: The owner cannot invoke products liability law to recover for a defect that causes
damage to, or the struction of, the product itself, and is instead left other protections he was able to obtain in
the K of sale via express or implied warranties.
 Tangible property damage can county as an injury

(b) What is a “product”?


 A PRODUCT is something typically something that is physical and material
 Not products (for the purposes of products liability)
o Real property (pre-made + mass produced homes may be considered products)
o human body parts, animals, textual materials (i.e. books), intangibles (i.e. electricity), and used
products
o Commercially sold “services”

68
(c) Who or what is a “seller”?
 MANUFACTURERS, DISTRIBUTORS, RETAILERS, AND COMMERCIAL SELLERS are all potential defendants
under strict products liability.
o These are the people who have control over the product and have a chance to inspect the goods for
defects before sale.
o These are people who are in the business of selling the product or products of that nature.
o Ex: A consumer injured by a defected lawnmower purchased at a local hardware store can sue the
store under a products liability theory because it sold the lawnmower
 Not sellers
o Casual sellers, sellers of services, and sellers of used goods are not defendants under strict products
liability.
 Sellers and strict liability
o b/c products liability applies to sellers, it often attaches liability to a person or entity that played no
role in the design, manufacture, or inspection of the product
 what constitutes selling
o selling= an actor taking steps to place the product on the market or figured in the distributional chain
through which the product is placed on the market
o an actual transfer or sale is not necessary to trigger products liability law, if the nature of the P’s
contact with the product is sufficiently close to the normal domain of the marketplace (actual sale or
transfer is not necessary)

(d) The key to products liability: defect


 US courts have identified 3 categories of defect: manufacturing defects, design defects, and warning (or
instructional defects) as well as tests as to what constitutes a defect

1. Manufacturing defect
o If it diverges form the manufacturer’s own specifications for the product

2. Design defect
o there is a flaw in the plan or specifications for the product. Flaws may be small or technical, or may
go to the essence of the product
o A design itself is defective when:
 (1) There are flaws in the blueprints, plans, etc.
 (2) Features that the product is supposed to have
 (3) A failure to warn about such defect or to instruct (i.e. mislabeled products)
o Some factors that come in to play when deciding whether something was defective is the risk of
physical injury posed by the particular design, how ordinary customers would expect the product to
function, and whether there is a feasible, safer, and affordable alternative design.
o To prevail on a claim of product defect, a plaintiff must be able to prove:
 (1) That the product was defective;
 (2) That the defect existed at the time that the product left the manufacturer’s hands; and
 (3) That the defect caused the plaintiff’s injury.

69
3. Failure to warn or instruct
o Defective when safety requires the product be sold with a warning but the product is sold without a
warning
o Gower v. Savage Arms., Inc. (E.D. Pa. 2001)
 Facts: This was the case about the guys who went hunting; one of them (Gower) was holding
the gun and accidentally shot himself in the foot. The defendant was a former soldier P
brought four claims:
 (1) The gun was not accompanied with warnings
 (2) The gun was defectively designed in that it could not be unloaded while in the
“safe” position
 (3) The gun was defectively designed in that it did not incorporate a detent system,
making the safety mechanisms more “user friendly”
 (4) The gun was defectively manufactured with a metal ridge that caused the gun’s
safety mechanism to fail over time
 Held: with respect to the different claims
 (1) no evidence that the gun lacked warning labels when it left the manufacturer
o Also noted that with his background in the military, he admitted that he
would not have acted differently if he did have the warnings
 (2), the court dismissed it, stating that the shooting did not happen during the
unloading process
o This was a causation issue – it makes the “unloading” factor irrelevant
 (3) there was insufficient evidence – the defendant did not provide evidence to
explain the lack of the detent, and the witness’ statements were unclear in regard to
the claim
 (4) held it valid and denied defendant’s motion for summary judgment
o Asserted that a gun of this type would not have a metal ridge that affects its
safety functions
 Notably, Gower need not show that the manufacturer was careless in making the gun

70

You might also like